Detailed Answers of UPSC Prelims 2021 GS Paper

These questions are from the set B of UPSC Civil Services Prelims Examination (2021) held on 10th of October. Our team has done tremendous hard work to bring the relevant and detailed answers to these questions. Students can refer to these detailed answers for their marks evaluation in Civil Services. UPSC CSE Answer Key 2021 will help the applicants to know their correct and incorrect responses and they can calculate the marks that they are going to score in the Prelims exam. The official UPSC Prelims Answer Key will be uploaded on the official website after the recruitment process is over. Till then, you can have a look at the detailed UPSC Prelims Question Paper Answers prepared by our experts which has been updated below.

Q1. With reference to the history of ancient India, Bhavabhuti, Hastimalla and Kshemeshvara were famous 

A. Jain Monks

B. Playwrights

C. Temple Architects

D. Philosophers

Answer: B

Bhavabhuti was an 8th-century scholar of India noted for his plays and poetry, written in Sanskrit. His plays are considered equal to the works of Kalidasa.

Hastimalla wrote 8 plays including VikrantKaurava and Subhadra Harana. He was a noted Kannada poet and playwright in the Hoysala Empire.

Q2. Consider the following statements: (Tikdam)

1. The Montagu Chelmsford Reforms of 1919 recommended granting voting rights to all women above the age of 21.

2. The Government of India Act of 1935 gave women reserved seats in the legislature.

Which of the statements given above is/are correct?

A. 1 only

B. 2 only

C. Both 1 and 2 

D. Neither 1 nor 2

Answer: B

The 1st statement is wrong. When the Montagu–Chelmsford Reforms were introduced in 1918 no recommendation was made for Indian women’s enfranchisement. Suffragists were active in drawing up petitions and published updates about the struggle in Stri Dharma, urging support for women’s political empowerment as a part of the anti-colonial movement against Britain. The Act did not grant women suffrage but included a clause that Indian provinces could enfranchise women if they chose to do so. It limited suffrage, barring most of India’s middle class, as it restricted the vote to those who had an annual income of more than ₹10,000–20,000; land revenues in excess of ₹250–500 per annum; or those recognised for their high level of public work or scholarship. Furthermore, it did not allow women to stand in elections. The law empowered the Imperial Legislative Assembly and the Council of State to grant the right to vote in those provinces in which legislative franchise had been approved, but the British Parliament retained the right to determine who could stand as candidates for the Legislative Councils.

The second statement is correct. The Government of India Act, 1935, as finally adopted, made twenty-nine million men and six million women eligible to vote. Seats were reserved for women on a communal basis while women could contest from any general seat as well.

Q3. With reference to 8th August 1942 in Indian History, which one of the following statements is correct?

A. The Quit India Resolution was adopted by the AICC.

B. The Viceroy’s Executive Council was expanded to include more Indians

C. The Congress ministries resigned in seven provinces.

D. Cripps proposed an Indian Union with full Dominion Status once the Second World War was over.

Answer: A

It’s an easy question. On August 8, 1942, the Quit India Resolution was passed at the Bombay session of the All India Congress Committee (AICC). On the same day, Gandhi urged Indians to act as a free nation and follow non-violent civil disobedience. This took place in August Kranti Maidan.

On the other hand, on 8 August 1940, the Viceroy Lord Linlithgow made a proposal called the August Offer which expanded the Executive Council to include more Indians. 

The Congress ministries resigned in October and November 1939, in protest against Viceroy Lord Linlithgow’s action of declaring India to be belligerent in the Second World War without consulting the Indian people.

The Cripps Mission happened in the months of March and April of 1942.

Q4. Who among the following is associated with ‘Songs from Prison’, a translation of Ancient Indian religious lyrics in English?

A Bal Gangadhar Tilak

B. Jawaharlal Nehru

C Mohandas Karamchand Gandhi

D. Sarojini Naidu

Answer: C

‘Songs from Prison’ was a translation from Sanskrit hymns and lyrics, from the Upanishads and other scriptures. Gandhi made these translations during his incarceration in Yeravda Prison in 1930. The other editor was John S Hoyland.

Q5. With reference to Medieval India, which one of the following is the correct sequence in ascending order in terms of size?

A. Pargana-Sarkar-Suba

B. Sarkar-Paragana-Suba

C. Suba-Sarkar-Pargana

D. Pargana-Suba-Sarkar

Answer: A

The Mughal Empire was essentially military in nature where the word of the emperor was law. The administrative structure was highly centralized as viewed by historians like Irfan Habib, Athar Ali, etc. Mughal Empire was divided into Suba; Suba was further divided into Sarkar, and Sarkar was divided further into Pargana and Pargana had various Villages under them. 

Q6. Who among the following was associated as Secretary with Hindu Female School which later came to be known as Bethune Female School?

A. Annie Besant

B. Debendra Nath Tagore

C. Ishwar Chandra Vidyasagar

D. Sarojini Naidu

Answer: C

This year, on September 4th, this institution was in the news when a bust of Ishwar Chandra Vidyasagar was unveiled in the institute. The college owes its origin to John Elliot Drinkwater Bethune. What began as Hindu Female School in 1849 was renamed Bethune School in 1856. In 1856, the Government took charge of the Hindu Female School, later renamed Bethune School. The Managing Committee of the school was then formed and Pandit Ishwar Chandra Vidyasagar was made the Secretary. In August 1878, Bethune School was amalgamated with Banga Maha Vidyalaya which was founded by Miss Annette Akroyd with the help of Durgamohan Das, Dwarka Nath Ganguly and Anandamohan Basu

Q7. In the context of Colonial India, Shah Nawaz Khan, Prem Kumar Sehgal and Gurbaksh Singh Dhillon are remembered as

A. Leaders of Swadeshi and Boycott Movement

B. Members of the Interim Government in 1946.

C. Members of the Drafting Committee in the Constituent Assembly.

D. Officers of the Indian National Army.

Answer: D

Quite an easy question once again. You must have read about the famous Red Fort Trials. Major General Shah Nawaz Khan, Colonel Gurbaksh Singh Dhillon and Colonel Prem Kumar Sahgal were three of the senior-most officers of INA and trusted colleagues of Netaji. They underwent court-martial by the British at Red Fort in 1945 and were sentenced to death, but instead had to be released following widespread protests and unrest in India. Congress leader and the country’s first Prime Minister, Jawaharlal Nehru, also got on board the INA officers’ legal defence team, along with party colleague Bhulabhai Desai and barrister Tej Bahadur Sapru.

Q8.  With reference to Indian history, which of the following statements is/are correct?

  1. The Nizamat of Arcot emerged out of Hyderabad state.
  2. The Mysore Kingdom emerged out of Vijayanagara Empire.
  3. The Rohilkhand Kingdom was formed out of the territories occupied by Ahmad Shah Durrani.

Select the correct answer using the code given below:

A. 1 and 2 only

B. 2 only

C. 2 and 3 only

D. 3 only

Answer: A

The Nawabs of the Carnatic (also referred to as the Nawabs of Arcot) were the nawabs who ruled the Carnatic region of South India between about 1690 and 1855. The Carnatic was a dependency of Hyderabad Deccan, and was under the legal purview of the Nizam of Hyderabad, until their demise. 

The second statement is correct. The Mysore kingdom, which was founded and ruled for the most part by the Hindu Wodeyar family, initially served as a vassal state of the Vijayanagara Empire. After the decline of the Vijayanagar Empire, Mysore became an independent state under the Hindu Wodeyar Dynasty in AD 1565. 

Regarding the third statement, it is incorrect because Nawab Ali Mohammed Khan, a scion of the ancient Barha Dynasty became the first Nawab of Rohilkhand, having been previously elected as overlord by various Afghan Chiefs at the age of fourteen. He would carve out the future kingdom from the collapsing Mughal Empire and go on to found the Rohilla Dynasty. The crown would go on being held by the Rohillas until the kingdom came to an end in 1774, and thereafter the same dynasty would rule over Rampur. Most of Rohilkhand’s borders were established by Ali Mohammed Khan and largely came into existence as a check to the power of the Oudh State.

Ahmad Shah Durrani was the founder of the Durrani Empire and is regarded as the founder of the modern state of Afghanistan. Within a few years, he extended his control from Khorasan in the west to Kashmir and North India in the east, and from the Amu Darya in the north to the Arabian Sea in the south.

Q9. Which one of the following statements is correct?

A. Ajanta Caves lie in the gorge of the Waghora river.

B. Sanchi Stupa lies in the gorge of the Chambal river.

C. Pandu-lena Cave Shrines lie in the gorge of the Narmada River.

D. Amravati Stupa lies in the gorge of the Godavari river.

Answer: A

The first statement is correct. The Ajanta caves are cut into the side of a cliff that is on the south side of a U-shaped gorge on the small river Waghora. 

The second statement is wrong. Sanchi Stupa lies in an upland plateau region, on a flat-topped sandstone hill, just west of the Betwa River and about 5 miles (8 km) southwest of Vidisha.

The third statement is wrong. The Buddhist monuments Pandavleni Caves also known by the names Pandu Lena caves and Trirashmi caves are a group of 24 rock-cut caves. It is located high up on the north face of Trivashmi hill 8 km south of Nasik city. The donative inscriptions indicate that these excavations are belonging to the Satavahana and Kshatrapa periods, spanning 2 C BC to the 3C AD. Buddha images were added to many of the caves in 5C-6C. The nearest river is the Godavari.

The fourth statement is wrong. Amravati stupa was founded by an emissary of Emperor Ashoka and its construction got completed around 2000 years ago. Amravati Stupa illustrates Lord Buddha in a human form and He is shown subduing an elephant. It is taller than the Sanchi stupa and has 95 ft high platforms, extending in the four cardinal directions, along with a huge circular dome. It is situated near river Krishna.

Q10. Consider the following statements:

  1. 21st February is declared to be International Mother Language Day by UNICEF.
  2. The demand that Bangla has to be one of the national languages was raised in the Constituent Assembly of Pakistan.

Which of the above statements is/are correct?

A. 1 only

B. 2 only

C. Both 1 and 2

D. Neither 1 nor 2

Answer: C

The first statement is incorrect because of the UNESCO/UNICEF angle which we will come to later. The Constituent Assembly of Pakistan was in session at Karachi-then the capital of Pakistan-from 23 February 1948. It was proposed that the members would have to speak either in Urdu or in English at the Assembly. Dhirendra Nath Datta, a member of the East Pakistan Congress Party, moved an amendment motion to include Bangla as one of the languages of the Constituent Assembly.  In the same year, the Government of the Dominion of Pakistan ordained Urdu as the sole national language, sparking extensive protests among the Bengali-speaking majority of East Bengal. Facing rising sectarian tensions and mass discontent with the new law, the government outlawed public meetings and rallies. The students of the University of Dhaka and other political activists defied the law and organised a protest on 21 February 1952. The movement reached its climax when police killed student demonstrators on that day. The deaths provoked widespread civil unrest. After years of conflict, the central government relented and granted official status to the Bengali language in 1956. In Bangladesh, 21 February (Ekushey February) is observed as Language Movement Day, a national holiday. In 1999, UNESCO declared 21 February as International Mother Language Day, in tribute to the Language Movement and the ethnolinguistic rights of people around the world.

Q11. With reference to Chausath Yogini Temple situated near Morena, consider the following statements:

  1. It is a circular temple built during the reign of the Kachchhapaghata Dynasty.
  2. It is the only circular temple built in India.
  3. It was meant to promote the Vaishnava cult in the region.
  4. Its design has given rise to a popular belief that it was the inspiration behind the Indian Parliament building.

Which of the statements given above are correct?

A. 1 and 2 only

B. 2 and 3 only

C. 1 and 4 only

D. 2, 3 and 4 only

Answer: C

Tikdam is easily applicable here. With a little bit of knowledge, you know that statement 4 is correct. On the other hand, the 2nd statement is too extreme. That gives us the answer which is 1 and 4 only.

According to an inscription, the temple was constructed by Maharaja Devapala of the Kacchapagata dynasty, dated VS 1380 (1323 CE). This temple is situated on top of a small hill and shows a circular plan. The temple is built on a high plinth and shows pillared cloisters that run around the wall facing an open courtyard. The small cells that form 64 subsidiary shrines have a shallow pillared mandapa in front; while a circular main shrine facing east stands in the middle of the courtyard. The cells and the main shrine are flat-topped, but it is believed that initially, each had a shikhara on top. While the 64 Yoginis originally placed in the 64 subsidiary shrines are now missing, a Shiva linga has taken their places in each cell. 

Q12. Which one of the following ancient towns is well-known for its elaborate system of water harvesting and management by building a series of dams and channelizing water into connected reservoirs?

A. Dholavira

B. Kalibangan

C. Rakhigarhi

D. Ropar

Answer: A

The site was in news quite recently and it is expected that you know this. Dholavira is located on Kadir island in the Rann of Kutch in Gujarat. The city had an impressive and unique water harvesting and management system. It can be noted that this area receives less than 160 cm of rain every year and is very prone to droughts.  A sophisticated water management system demonstrates the ingenuity of the Dholavira people in their struggle to survive and thrive in a harsh environment.

Q13. In the first quarter of the seventeenth century, in which of the following was/were the factory/factories of the English East India Company located?

  1. Broach
  2. Chicacole
  3. Trichinopoly

Select the correct answer using the code given below:

A. 1 only

B. 1 and 2 only

C. 3 only

D. 2 and 3 only

Answer: A

By 1623, the English East India Company had established factories at Surat, Broach, Ahmedabad, Agra, and Masulipatam. 

Q14. From the decline of Guptas until the rise of Harshavardhana in the early seventh century, which of the following kingdoms were holding power in Northern India?

  1. The Guptas of Magadha
  2. The Paramaras of Malwa
  3. The Pushyabhutis of Thanesar
  4. The Maukharis of Kanauj
  5. The Yadavas of Devagiri
  6. The Maitrakas of Valabhi

Select the correct answer using the code given below.

A. 1, 2 and 5

B. 1, 3, 4 and 6

C. 2, 3 and 4

D. 5 and 6

Answer: B

The Later Gupta dynasty ruled the Magadha region in eastern India between the 6th and 7th centuries CE. The Later Guptas succeeded the imperial Guptas as the rulers of Magadha. So 1st is correct. The Paramaras of Malwa rose on the ruins of the Pratihara empire of Kanauj in 11 the Century. So statement 2 is incorrect.  The Maukharis (554 CE – 606 CE) rose as a power after the downfall of the Gupta Empire (3rd to 6th century CE) in the 6th century CE in northern India.  So 3rd is correct.

Q15. According to the Portuguese writer Nuniz, the women in the Vijayanagara Empire were expert in which of the following areas?

1. Wrestling

2. Astrology

3. Accounting

4. Soothsaying

Select the correct answer using the code given below.

A. 1, 2 and 3 only

B. 1, 3 and 4 only

C. 2 and 4 only

D. 1, 2, 3 and 4

Answer: D

All claims are correct.

Q16. With reference to Madanapalle of Andhra Pradesh, which one of the following statements is correct?

A. Pingali Venkayya designed the tricolour Indian National Flag here.

B. Pattabhi Sitaramaih led the Quit India Movement of Andhra Region from here.

C. Rabindranath Tagore translated the National Anthem from Bengali to English here.

D. Madame Blavatsky and Colonel Olcott set up the headquarters of Theosophical Society first here.

Answer: C 

Rabindranath Tagore translated “Jana Gana Mana” from Bengali to English and also set it to music in Madanapalle.

Q17. Consider the following pairs:

(Historical place) (Well-known for)

1. Burzahom : Rock-cut shrines

2. Chnadra-ketugarh : Terracotta art

3. Ganeshwar : Copper artefacts

Which of the pairs given above is/are correctly matched?

A. 1 only

B. 2 only

C. 3 only

D. 2 and 3

Answer: D

Chandraketugarh features many examples of terracotta art, displaying an unusual degree of precision and craftsmanship. These plaques are comparable to those found at other better-known sites such as Kaushambi and Ahichhatra. The terracotta plaques from these sites often carry similar motifs executed in nearly identical fashion. This similarity suggests an established communication link and common cultural heritage among these sites.

Ganeshwar had mainly supplied copper objects to Harappa. The archaeological site was named after the present-day village, Gilund, and is locally known as Modiya Magari which means “bald habitation mound”

Q18. Consider the following statements:

  1. It was during the reign of Iltutmish that Chengiz Khan reached the Indus in pursuit of the fugitive Khwarezm prince.
  2. It was during the reign of Muhammad bin Tughluq that Taimur occupied Multan and crossed the Indus.
  3. It was during the reign of Deva Raya 2 of Vijayanagara Empire that Vasco da Gama reached the coast of Kerala.

Which of the statements given above is/are correct?

A. 1 only

B 1 and 2

C. 3 only

D. 2 and 3

Answer: A

Q19. Consider the Following Statements:

1. St. Francis Xavier was one of the founding members of the Jesuit Order.

2. St. Francis Xavier died in Goa and a church is dedicated to him there.

3. The Feast of St. Francis Xavier is celebrated in Goa each year.

Which of the statements given above are correct?

A. 1 and 2 only

B. 2 and 3 only

C. 1 and 3 only

D. 1, 2 and 3

Answer: C

He died in China and hence once you eliminate this wrong statement using this Tikdam, you come to the right answer.

Q20. With reference to the history of ancient India, which of the following statements is/are correct?

1. Mitakshara was the civil law for upper castes and Dayabhaga was the civil law for lower castes.

2. In the Mitakshara system, the sons can claim right to the property during the lifetime of the father, whereas in the Dayabhaga system, it is only after the death of the father that the sons can claim right to the property.

3. The Mitakshara system deals with the matter related to the property held by male members only of a family, whereas the Dayabhaga system deals with the matters related to the property held by both male and female members of a family.

Select the correct answer using the code given below.

A. 1 and 2

B. 2 only

C. 1 and 3

D. 3 only

Answer: B

The Dāyabhāga is a Hindu law treatise written by Jīmūtavāhana which primarily focuses on inheritance procedure. The Dāyabhāga was the strongest authority in Modern British Indian courts in the Bengal region of India, although this has changed due to the passage of the Hindu Succession Act of 1956 and subsequent revisions to the act. Based on Jīmūtavāhana’s criticisms of the Mitākṣarā, it is thought that his work is precluded by the Mitākṣarā. This has led many scholars to conclude that the Mitākṣarā represents the orthodox doctrine of Hindu law, while the Dāyabhāga represents the reformed version

Q21. The money multiplier in an economy increases with which one of the following?

A. Increases in the Cash Reserve Ratio in the banks

B. Increases in the Statutory Liquidity Ratio in the banks

C. Increase in the banking habit of the people

D. Increases in the Population of the country

Answer: C

Q22. With reference to the Indian economy, demand-pull inflation can be caused/increased by which of the following?

1. Expansionary policies

2. fiscal stimulus

3. Inflation-indexing wages

4. Higher purchasing power

5. Rising interest rates

Select the correct answer using the code given below.

A. 1, 2 and 4 only

B. 3, 4 and 5 only

C 1, 2, 3 and 5 only

D. 1, 2, 3, 4 and 5

Answer: A

Q23. With reference to India, consider the following statements:

1. Retail investors through demat accounts can invest in ‘Treasury Bills’ and ‘Government of India Debt Bonds’ in the primary market.

2. The ‘Negotiated Dealing System Order Matching’ is a Government securities trading platform of the Reserve Bank of India.

3. The ‘Central Depository Services Ltd.’ Is jointly promoted by the Reserve Bank of India and the Bombay Stock Exchange.

Which of the statements given above is/are correct?

A. 1 only

B. 1 and 2

C. 3 only

D. 2 and 3

Answer: B

Promoters/Shareholders

CDSL is promoted by Bombay Stock Exchange Ltd (BSE Ltd) – Asia’s latest Stock Exchange. Other shareholders include HDFC Bank Ltd, Standard Chartered Bank and Canara Bank.

NDS-OM is a screen-based electronic anonymous order matching system for secondary market trading in Government securities owned by RBI. Presently the membership of the system is open to entities like Banks, Primary Dealers, Insurance Companies, Mutual Funds etc. i.e entities who maintain SGL accounts with RBI.

Q24. With reference to ‘WaterCredit’, consider the following statements:

1. It puts microfinance tools to work in the water and sanitation sector.

2. It is a global initiative launched under the aegis of the World Health Organization and the World Bank.

3. It aims to enable the poor people to meet their water needs without depending on subsidies.

Which of the statements given above are correct?

A. 1 and 2 only

B. 2 and 3 only

C. 1 and 3 only

D. 1, 2 and 3

Answer: C

The Water Credit Initiative has been funded by social entrepreneur Gary White and Hollywood Actor Matt Damon through their organization Water.org which has invested US$ 2.2 million in Water Credit programs. Water credit applies the principles of microfinance to the water and sanitation sector in developing countries. By making small loans to individuals and communities who do not have access to credit, Water-Credit empowers people to address their own water and sanitation needs instead of depending on government funds and charity.

Q25. In India, the central bank’s function as the ‘lender of last resort’ usually refers to which of the following?

1. Lending to trade and industry bodies when they fail to borrow from other sources

2. Providing liquidity to the banks having a temporary crisis

3. Lending to governments to finance budgetary deficits

Select the correct answer using the code given below:

A. 1 and 2

B. 2 only

C. 2 and 3

D. 3 only

Answer: B

Q26. ‘R2 code of practices’ constitutes a tool available for promoting the adoption of

A. environmentally responsible practices in electronics recycling industry

B. ecological management of ‘Wetlands of International importance’ under the Ramsar Convention

C. sustainable practices in the cultivation of agricultural crops in degraded lands

D. ‘Environmental Impact Assessment’ in the exploitation of natural resources

Answer: A

R2 means responsible recycling. R2 stands for Responsible Recycling and is a standard specifically created for the electronics recycling industry by Sustainable Electronics Recycling International (SERI). The standards were then accredited by ANAB, and in 2008, R2 was released.

Q27. Why there is a concern about copper smelting plants?

1. They may release lethal quantities of carbon monoxide into the environment.

2. The copper slag can cause the leaching of some heavy metals into the environment.

3. They may release sulphur dioxide as a pollutant.

Select the correct answer using the codes given below.

A. 1 and 2 only

B. 2 and 3 only

C. 1 and 3 only

D. 1, 2 and 3

Answer: B

The smelting of sulfide ores results in the emission of sulfur dioxide gas, which reacts chemically in the atmosphere to form a sulfuric acid mist.

Q28. With reference to furnace oil. Consider the following statements:

1. It is a product of oil refineries.

2. Some industries use it to generate power.

3. Its use causes sulphur emissions into the environment.

Which of the statements given above are correct?

A. 1 and 2 only

B. 2 and 3 only

C. 1 and 3 only

D. 1, 2 and 3

Answer: D

Q29. What is blue carbon?

A. Carbon captured by oceans and coastal ecosystems

B. Carbon sequestered in forest biomass and agricultural soils

C. Carbon contained in petroleum and natural gas

D. Carbon present in atmosphere

Answer: A

Q30. In the nature, which of the following is/are most likely to be found surviving on a surface without soil?

1. Fern

2. Lichen

3. Moss

4. Mushroom

Select the correct answer using the code given below.

A. 1 and 4 only

B. 2 only

C. 2 and 3

D. 1, 3 and 4

Answer: C

Q31. Consider the following statements:

1. The Governor of the Reserve Bank of India (RBI) is appointed by the Central Government.

2. Certain provisions in the Constitution of India give the Central Government the right to issue directions to the RBI in the public interest.

3. The Governor of the RBI draws his power from the RBI Act. 

Which of the above statements are correct?

A. 1 and 2 only

B. 2 and 3 only

C. 1 and 3 only.

D. 1, 2 and 3

Answer: C

The statement 1 is correct – the RBI Governor and Deputy Governors are appointed by the Central Government. Their names are cleared by the Cabinet Committee on appointments.

The statement 2 is wrong – According to Section 7 of the RBI Act the central government is empowered to issue directions it considers necessary for public interest to the central bank from time to time after consultation with the RBI governor. It is not the constitution.

Q32. With reference to casual workers employed in India, consider the following statements :

1. All casual workers are entitled to Employees Provident Fund coverage.

2. All casual workers are entitled for regular working hours and overtime payment.

3. The government can by a notification specify that an establishment or industry shall pay wages only through its bank account.

Which of the above statements are correct?

A. 1 and 2 only

B. 2 and 3 only 

C. 1 and 3 only

D. 1, 2 and 3

Answer: A 

Entitlements of the provident fund to be extended to contractual employees: Supreme Court

The Hon’ble Supreme Court of India, vide its judgment dated 17.01.2020 in the matter of M/s. Pawan Hans Limited & Ors. Vs. Aviation Karmachari Sanghatana & Ors. clarified that the contractual employees who are drawing wages/salary directly or indirectly are entitled to the Provident Fund Benefits.

The judgment by SC definitely brings a respite to the contractual employees of the Organization as well as other nationwide employees in such a situation, who had been seeking equality with the regular employees under the PF Trust Regulations of India.

https://www.greythr.com/blog/whether-contractual-employees-are-entitled-pf-benefits/

statement 1 is right

The Payment of Wages  Act, 1936 has been amended by Payment of Wages (Amendment) Act, 2017 (effective from 28.12.2016) to enable the employers to pay wages to their employees by (a) cash or (b) cheque or (c) crediting to their bank account. The amendment in the Act also enables the appropriate Government to specify the industrial or other establishment, by notification in the Official Gazette, which shall pay to every person employed in such industrial or other establishment the wages only by cheque or by crediting in his bank account.

https://pib.gov.in/newsite/PrintRelease.aspx?relid=187043

Statement 3 is wrong

Q33. Which among the following steps is most likely to be taken at the time of an economic recession?

A. Cut in tax rates accompanied by increase in interest rate 

B. Increase in expenditure on public projects

C. Increase in tax rates accompanied 

D. Reduction of expenditure on public projects

In case of Economic Recession, we need to increase the money supply into the economy.

Ways to boost money into the economy:

Cutting the tax rate

Reducing interest rate – easier available loans and credit

Increase the public expenditure

Answer – B

Q34. Consider the following statements: 

Other things remaining unchanged, market demand for a good might increase if

1. price of its substitute increases 

2. price of its complement increases

3. the good is an inferior good and the income of the consumers increases its price falls

4. Its prices fall

Which of the above statements are correct?

A. 1 and 4 only

B. 2, 3 and 4

C. 1, 3 and 4

D. 1, 2 and 3

Answer: C

When the price of a substitute for a good falls, the demand for that good will decline and when the price of the substitute rises, the demand for that good will increase.

The goods which are complementary with each other, the fall in the price of any of them would favorably affect the demand for the other.

The greater income means the greater purchasing power. Therefore, when incomes of the people increase, they can afford to buy more. It is because of this reason that increase in income has a positive effect on the demand for a good.

Q35. With reference to ‘Urban Cooperative Banks’ in India, consider the following statements : 

1. They are supervised and regulated by local boards set up by the State Governments. 

2. They can issue equity shares and preference shares. 

3. They were brought under the purview of the Banking Regulation Act, 1949 through an Amendment in 1966.

Which of the statements given above is/are correct?

A. 1 only

B. 2 and 3 only

C. 1 and 3 only

D. 1, 2 and 3

Answer: B 

Statement 1 is wrong

RBI is the one regulating them now.

Statement 2 – is correct

The Reserve Bank on Wednesday came out with draft guidelines allowing primary urban cooperative banks (UCBs) to augment capital through issuance of equity shares, preference shares and debt instruments.

https://www.business-standard.com/article/pti-stories/rbi-issues-draft-circular-on-allowing-ucbs-to-augment-capital-121071400994_1.html

Statement 3 is correct – They were brought under the RBI’s watch in 1966, a move that brought the problem of dual regulation along with it.

Q36. Indian Government Bond Yields are influenced by which of the following? 

1. Actions of the United States Federal Reserve 

2. Actions of the Reserve Bank of India 

3. Inflation and short-term interest rates

Select the correct answer using the code given below.

A. 1 and 2 only

B. 2 only

C. 3 only

D. 1, 2 and 3

Answer: D

The multifaceted roles played by the RBI in the payment system, monetary policy, financial stability policy, and policy coordination with the Treasury gives it the operational ability to influence government bonds’ nominal yields by setting and changing the short-term interest rate and using other tools of monetary policy as it deems appropriate.

Statement 2 – correct

https://www.adb.org/sites/default/files/publication/488961/adr-vol36no1-7-government-bond-yields-india.pdf

Statement 3 is also correct

Q37. Consider the following:

1. Foreign currency convertible bonds 

2. Foreign institutional investment with certain conditions 

3. Global depository receipts 

4. Non-resident external deposits

Which of the above can be included in Foreign Direct Investments?

A. 1, 2 and 3

B. 3 only

C. 2 and 4

D. 1 and 4

Answer: D

FIIs can include hedge funds, insurance companies, pension funds, investment banks, and mutual funds. FIIs can be important sources of capital in developing economies, yet many developing nations, such as India, have placed limits on the total value of assets an FII can purchase and the number of equity shares it can buy, particularly in a single company.3 This helps limit the influence of FIIs on individual companies and the nation’s financial markets, and the potential damage that might occur if FIIs fled en masse during a crisis.

https://www.investopedia.com/terms/f/fii.asp

Option 2 is wrong

A global depositary receipt (GDR) is a certificate issued by a bank that represents shares in a foreign stock on two or more global markets.

GDRs typically trade on American stock exchanges as well as Eurozone or Asian exchanges.

GDRs and their dividends are priced in the local currency of the exchanges where the shares are traded.

GDRs represent an easy, liquid way for U.S. and international investors to own foreign stocks.

https://www.investopedia.com/terms/g/gdr.asp

Option 3 is wrong

Q38. Consider the following statements:

The effect of devaluation of a currency is that it necessarily

1. Improves the competitiveness of the domestic exports in the foreign markets

2. increases the foreign value of domestic currency 

3. improves the trade balance 

Which of the above statements is/are correct?

A. 1 only

B. 1 and 2

C. 3 only

D. 2 and 3

https://www.economicshelp.org/macroeconomics/exchangerate/effects-devaluation/

A devaluation of the exchange rate will make exports more competitive and appear cheaper to foreigners. This will increase demand for exports. 

statement 1 is correct.

A devaluation in the Pound means £1 is worth less compared to other foreign currencies. For example

Jan 2016. £1= $1.50

July 2016 – £1=$1.28

statement 2 is wrong.

Statement 3 not necessary – petrol

Answer – A

Q39. Which one of the following effects of the creation of black money in India has been the main cause of worry to the Government of India?

A. Diversion of resources to the purchase of real estate and investment in luxury housing 

B. Investment in unproductive activities and purchase of precious stones, jewellery, gold, etc. 

C. Large donations to political parties and growth of regionalism

D. Loss of revenue to the State Exchequer due to tax evasion

Tax evasion is the greatest issue.

Answer – D

Q40. Which one of the following is likely to be the most inflationary in its effects?

A. Repayment of public debt

B. Borrowing from the public to finance a budget deficit

C. Borrowing from the banks to finance a budget deficit

D. Creation of new money to finance a budget deficit

If you print more money, the amount of goods doesn’t change. However, if you print money, households will have more cash and more money to spend on goods. If there is more money chasing the same amount of goods, firms will just put up prices.

https://www.economicshelp.org/blog/797/economics/why-printing-money-causes-inflation/
https://www.business-standard.com/article/economy-policy/explained-why-poorer-nations-can-t-just-print-more-money-and-become-rich-118121900185_1.html

Answer – D

Q41. Which one of the following is used in preparing a natural mosquito repellent?

A. Congress grass

B. Elephant grass

C. Lemongrass

D. Nut grass

Answer – C

Lemon grass

Known for its distinct smell, citronella grass (or lemon grass) is the most commonly used natural ingredient in mosquito repellants.

https://www.gardendesign.com/plants/mosquito-repellent.html

Q42. Consider the following kinds of organisms:

  1. Copepods
  2. Cyanobacteria
  3. Diatoms
  4. Foraminifera

Which of the above are primary producers in the food chains of oceans?

A. 1 and 2

B. 2 and 3

C. 3 and 4

D. 1 and4

Foraminifera are single-celled organisms, members of a phylum or class of amoeboid protists characterized by streaming granular ectoplasm for catching food and other uses; and commonly an external shell of diverse forms and materials. 

Copepods – It is a primary consumer

Diatoms and cyanobacteria are producers

Answer – B

Q43. Consider the following animals:

  1. Hedgehog
  2. Marmot
  3. Pangolin

To reduce the chance of being captured by predators, which of the above organisms roll up and protect its vulnerable parts?

A. 1 and 2

B. 2 only

C. 3 only

D. 1 and 3

Answer – D 

These solitary, primarily nocturnal animals, are easily recognized by their full armor of scales. A startled pangolin will cover its head with its front legs, exposing its scales to any potential predator. If touched or grabbed it will roll up completely into a ball, while the sharp scales on the tail can be used to lash out. 

https://www.worldwildlife.org/species/pangolin

Option 3 – correct

Hedgehogs have a coat of stiff, sharp spines. If attacked they will curl into a prickly and unappetizing ball that deters most predators. 

https://www.nationalgeographic.com/animals/mammals/facts/hedgehog

Option 1 – correct

Q44. With reference to the ‘New York Declaration on Forests’, which of the following statements are correct?

1. It was first endorsed at the United Nations Climate Summit in 2014.

2. It endorses a global timeline to end the loss of forests.

3. It is a legally binding international declaration.

4. It is endorsed by governments, big companies and indigenous communities.

5. India was one of the signatories at its inception.

Select the correct answer using the code given below.

A. 1, 2 and 4

B. 1, 3 and 5

C. 3 and 4

D. 2 and 5

Answer – A

The New York Declaration on Forests is a voluntary and non-legally binding political declaration which grew out of dialogue among governments, companies and civil society, spurred by the United Nations Secretary-General’s Climate Summit held in New York in 2014. The Declaration pledges to halve the rate of deforestation by 2020, to end it by 2030.

Q45. Magnetite particles, suspected to cause neurodegenerative problems, are generated as environmental pollutants from which of the following?

1. Brakes of motor vehicles

2. Engines of motor vehicles

3. Microwave stoves within homes

4. Power plants

5. Telephone lines

Select the correct answer using the code given below:

A. 1, 2, 3 and 5 only

B. 1, 2 and 4 only

C. 3, 4 and 5 only

D. 1, 2, 3, 4 and 5

Answer: D

Magnetic nanoparticles, also called as PM, are a class of nanoparticle that can be manipulated using magnetic fields. Such particles commonly consist of two components, a magnetic material, often iron, nickel and cobalt, and a chemical component that has functionality. They are fine dust emitted by metallurgical industry, vehicle brake systems are the major source of airborne magnetite at the roadside, diesel- and petrol-engine exhaust PM, power plants.

Q46. Which of the following is a filter feeder?

A. Catfish 

B. Octopus

C. Oyster

D. Pelican

Answer: C

Explanation: Filter feeders are a subgroup of suspension feeding animals that feed by straining suspended matter and food particles from water, typically by passing the water over a specialized filtering structure. Some animals that use this method of feeding are clams, krill, sponges, baleen whales, and many fish (including some sharks). Some birds, such as flamingos and certain species of duck, are also filter feeders. 

Although other animals at the reef are said to filter-feed, the method used by sponges is unique to this simple group of animals. In fact, sponges are named because of the many pores covering their bodies. They belong to the Phylum Porifera and may be called poriferans.

https://en.wikipedia.org/wiki/Filter_feeder
https://www.coris.noaa.gov/activities/resourceCD/resources/sponge_lp.pdf

Q47. In case of which one of the following biogeochemical cycles, the weathering of rocks is the main source of release of nutrient to enter the cycle?

A. Carbon cycle

B. Nitrogen cycle

C. Phosphorus cycle

D. Sulphur cycle

Answer: C

Explanation: In nature, phosphorus is found mostly in the form of phosphate ions. Phosphate compounds are found in sedimentary rocks, and as the rocks weather—wear down over long time periods—the phosphorus they contain slowly leaches into surface water and soils. Volcanic ash, aerosols, and mineral dust can also be significant phosphate sources, though phosphorus has no real gas phase, unlike other elements such as carbon, nitrogen, and sulfur.

https://www.britannica.com/science/phosphorus-cycle

Q48. Which of the following are detritivores?

  1. Earthworms
  2. Jellyfish 
  3. Millipedes
  4. Seahorses
  5. Woodlice

Select the correct answer using the codes given below.

A. 1,2 and 4 only

B. 2,3,4 and 5 only

C. 1,3 and 5 only

D. 1,2,3,4 and 5

Answer: C

Explanation: A detritivore is a heterotrophic organism, which obtains its nutrition by feeding on detritus. Detritus is an organic matter made up of dead plant and animal material. Detritivores may also obtain nutrition by coprophagy, which is a feeding strategy involving the consumption of faeces.

Ocean decomposers have a variety of methods for gathering dead material to feed on. Echinoderms like sea urchins, sea stars and sea cucumbers hunt and eat live food, but they also move around and consume decaying organic matter that covers rocks and other surfaces.

Detritivores are often invertebrate insects such as mites, beetles, butterflies and flies; molluscs such as slugs and snails; or soil-dwelling earthworms, millipedes and woodlice. 

Q49. The ‘Common Carbon Metric’ supported by UNEP, has been developed for

A. assessing the carbon footprint of building operations around the world

B. enabling commercial farming entities around the world to enter carbon emission trading

C. enabling governments to assess the overall carbon footprint caused by their countries

D. assessing the overall carbon footprint caused by the use of fossil fuels by the world in a unit time.

Answer: A

Explanation: The Common Carbon Metric is the calculation used to define the measurement, reporting, and verification for GHG emissions associated with the operation of building types of particular climate regions. It does not include value-based interpretation of the measurements such as weightings or benchmarking.

https://c2e2.unepdtu.org/wp-content/uploads/sites/3/2016/09/unep-sbci-common-carbon-metric.pdf

Q50.  Which of the following have species that can establish symbiotic relationship with other organisms?

  1. Cnidarians
  2. Fungi
  3. Protozoa

Select the correct answer using the code given below 

A. 1 and 2 only

B. 2 and 3 only

C. 1 and 3 only

D. 1,2 and 3

Answer: D

Explanation: The symbiotic association between the invertebrate phylum Cnidaria (Coelenterate) and the unicellular dinoflagellate algae, called zooxanthellae, is very common. The most well-known relationship is between zooxanthellae and hermatypic, or reef-forming, corals.

Mycorrhizae are symbiotic relationships that form between fungi and plants. The fungi colonize the root system of a host plant, providing increased water and nutrient absorption capabilities while the plant provides the fungus with carbohydrates formed from photosynthesis.

Mutualism, where two species benefit from their interaction. Some scientists believe that these are the only true examples of symbiosis. For example, termites have a mutualistic relationship with protozoa that live in the insect’s gut. The termite benefits from the ability of bacterial symbionts within the protozoa to digest cellulose. The termite itself cannot do this, and without the protozoa, it would not be able to obtain energy from its food (cellulose from the wood it chews and eats).

Q51. Under the Indian Constitution, concentration of wealth violates

A. The Right to Equality

B. Directive Principles of State Policy

C. the Right to Freedom

D. the Concept of Welfare 

Answer: B

Explanation: Article 39 of the Indian Constitution requires the State to direct its policies towards securing for all its citizens the right to an adequate means of livelihood, availability of material resources, and ensuring prevention of concentration of wealth, etc.

https://www.constitutionofindia.net/constitution_of_india/directive_principles_of_state_policy/articles/Article%2039

Q52. What is the position of Right to Property in India?

A. Legal right available to citizens only

B. Legal right available to any person

C. Fundamental Right available to citizens only

D. Neither Fundamental Right nor Legal Right

Answer: A

Explanation: A citizen’s right to own private property is a human right. The state cannot take possession of it without following due procedure and authority of law, the Supreme Court has held in a judgment.

The state cannot trespass into the private property of a citizen and then claim ownership of the land in the name of ‘adverse possession’, the court said.

https://www.thehindu.com/news/national/private-property-is-a-human-right-says-sc/article30551819.ece

Q53. What was the exact Constitutional status of India on 26th January, 1950?

A. A Democratic Republic

B. A Sovereign Democratic Republic 

C. A Sovereign Secular Democratic Republic

D. A Sovereign Secular Socilaist Democratic Republic

Answer: B

Q54.  Constitutional government means

A. A representative government of a nation with federal structure

B. A government whose Head enjoys nominal powers

C. A government whose Head enjoys real powers

D. A government limited by the terms of the Constitution

Answer: D

Q55. With reference to India, the terms ‘Halbi, Ho and Kui’ pertain to

A. Dance forms of Northwest India

B. Musical instruments

C. Prehistoric cave paintings

D. Tribal languages

Answer: D

Explanation: Can be solved by Tikdam. ‘Ho’ is a famous tribal language. 

Q56. Consider the following statements in respect of Bharat Ratna and Padma Awards:

1. Bharat Ratna and Padma Awards are titles under the Article 18(1) of the Constitution

 of India

2. Padma Awards, which were instituted in 1954, were suspended only once.

3. The number of Bharat Ratna Awards is restricted to a maximum of five in a particular   

year.

Which of the above statements are not correct?

A. 1 and 2 only

B. 2 and 3 only

C. 1 and 3 only

D. 1, 2 and 3

Answer: d

Explanations: The Bharat Ratna is the highest civilian award of the Republic of India. Instituted on 2 January 1954, the award is conferred in recognition of exceptional service/performance of the highest order, without distinction of race, occupation, position, or sex. The award was originally limited to achievements in the arts, literature, science, and public services, but the government expanded the criteria to include “any field of human endeavour” in December 2011. The recommendations for the Bharat Ratna are made by the Prime Minister to the President, with a maximum of three nominees being awarded per year. 

The Bharat Ratna, along with other personal civil honours, was briefly suspended from July 1977 to January 1980, during the change in the national government; and for a second time from August 1992 to December 1995, when several public-interest litigations challenged the constitutional validity of the awards. 

Q57. Consider the following statements:

  1. Statement 1: The United Nations Capital Development Fund (UNCDF) and the Arbor Day Foundation have recently recognized Hyderabad as 2020 Tree City of the World
  2. Statement 2: Hyderabad was selected for the recognition for a year following its commitment to grow and maintain the urban forests.

Which one of the following is correct in respect of the above statements?

A. Both Statement 1 and Statement 2 are correct and Statement 2 is the correct explanation for Statement 1

B. Both Statement 1 and Statement 2 are correct but Statement 2 is not the correct explanation for Statement 1

C. Statement 1 is correct but Statement 2 is not correct

D. Statement 1 is not correct but Statement 2 is correct

Answer: D

Explanation: In a unique distinction, Hyderabad has become the only city in India to be recognised as a ‘Tree City of the World’ by the Arbor Day Foundation and the Food and Agriculture Organization (FAO) of the United Nations. Hyderabad is placed alongside 119 other cities from 63 countries.

The countries have been recognised for their commitment to growing and maintaining urban forests in building healthy, resilient and happy cities.

Q58. Consider the following statements in respect of the Laureus World Sports Award which was instituted in the year 2000:

1. American golfer Tiger Woods was the first winner of this award.

2. The award was received mostly by ‘Formula One’ players so far.

3. Roger Federer received this award maximum number of times compared to others.

Which of the above statements are correct?

A. 1 and 2 only

B. 2 and 3 only

C. 1 and 3 only

D. 1, 2 and 3

Answer: C

Explanation: Swiss tennis player Roger Federer holds the record for the most awards with six, five for Sportsman of the Year and one for Comeback of the Year. 

The inaugural winner of the award was the American golfer Tiger Woods who finished the 1999 season with eight wins, a feat not achieved since 1974, including the PGA Championship.

Formula One has won 4 awards while tennis has a maximum of 11 awards. 

Q59. Consider the following statements in respect of the 32nd Summer Olympics:

1. The official motto for this Olympics is ‘A New World’.

2. Sport Climbing, Surfing, Skateboarding, Karate and Baseball are included in this Olympics.

Which of the above statements is/are correct?

A. 1 only

B. 2 only

C. Both 1 and 2

D. Neither 1 nor 2

Answer: D

Explanation:  The disciplines added by the Japanese Olympic Committee were baseball and softball, karate, sport climbing, surfing and skateboarding, the last four of which made their Olympic debuts, and the last three of which will remain on the Olympic program.

Q60. Consider the following statements in respect of the ICC World Test Championship:

1. The finalists were decided by the number of matches they won.

2. New Zealand was ranked ahead of England because it won more matches than England.

Which of the above statements is/are correct?

A. 1 only

B. 2 only

C. Both 1 and 2

D. Neither 1 nor 2

Answer: D

Q61. Consider the following statements:

1. ‘Right to the City’ is an agreed human right and the UN-Habitat monitors the commitments made by each country in this regard.

2. ‘Right to the City’ gives every occupant of the city the right to reclaim public spaces and public participation in the city.

3. ‘Right to the City’ means that the State cannot deny any public service or facility to the unauthorized colonies in the city.

Which of the statements given above is/are correct?

A. 1 only

B. 3 only

C. 1 and 2

D. 2 and 3

Answer: A

Explanation: 

Right to the city has gained international recognition in the last years could be seen in the United Nations’ Habitat III process, and how the New Urban Agenda (2016) recognized the concept as the vision of “cities for all”

The right to the city is far more than the individual liberty to access urban resources: it is a right to change ourselves by changing the city. It is, moreover, a common rather than an individual right since this transformation inevitably depends upon the exercise of a collective power to reshape the processes of urbanization.

Q62. With reference to India, consider the following statements:

1. Judicial custody means an accused is in the custody of the concerned magistrate and such accused is locked up in a police station, not in jail.

2. During judicial custody, the police officer in charge of the case is not allowed to interrogate the suspect without the approval of the court.

Which of the statements given above is/are correct?

A. 1 only

B. 2 only

C. Both 1 and 2

D. Neither 1 nor 2

Answer: B

Explanation:

Police Custody means that police has the physical custody of the accused while Judicial Custody means an accused is in the custody of the concerned Magistrate. In the former, the accused is lodged in a police station lockup while in the latter, it is the jail.

During Judicial Custody,  the police officer in charge of the case is not allowed to interrogate the suspect. However, the court may allow the interrogations to be conducted if it opines the interrogation being necessary under the facts produced before the court.

Source: https://districts.ecourts.gov.in/sites/default/files/fct.pdf

Q63: With reference to India, consider the following statements:

1. When a prisoner makes out a sufficient case, parole because it becomes a matter of his/her right.

2. State Governments have their own Prisoners Release on Parole Rules.

Which of the statements given above is/are correct?

A. 1 only

B. 2 only

C. Both 1 and 2

D. Neither 1 nor 2

Answer: B

Explanation:

Furlough is seen as a matter of right, to be granted periodically irrespective of any reason and merely to enable the prisoner to retain family and social ties, parole is not a matter of right and may be denied to a prisoner even when he makes out a sufficient case.

Prisons are a State subject: each state has different rules and regulations for it.

Q64. At the national level, which ministry is the nodal agency to ensure effective implementation of the Scheduled Tribes and Other Traditional Forest Dwellers (Recognition of Forest Rights) Act, 2006?

A. Ministry of Environment, Forest and Climate Change

B. Ministry of Panchayat raj

C. Ministry of Rural Development

D. Ministry of Tribal Affairs

Answer – D:

Q65. A legislation which confers on the executive or administrative authority an unguided and uncontrolled discretionary power in the matter of application of law violates which one of the following Articles of the Constitution of India?

A. Article 14

B. Article 28

C. Article 32

D. Article 44

Answer: A

Article 14 of the Constitution of India provides for equality before the law or equal protection of the laws within the territory of India. … “The State shall not deny to any person equality before the law or the equal protection of the laws within the territory of India.” 

Q66: Which one of the following in Indian polity is an essential feature that indicates that it is federal in character?

A. The independence of the judiciary is safeguard.

B. The Union Legislature has elected representatives from constituent units.

C. The Union Cabinet can have elected representatives from regional parties.

D. The Fundamental Rights are enforceable by Courts of Law.

Answer: A

Q67:  Which one of the following best defines the term ‘State’?

A. A community of persons permanently occupying a definite territory independent of external control and possessing an organized government.

B. A politically organized people of a definite territory and possessing an authority to govern them, maintain law and order, protect their natural rights and safeguard their means of sustenance.

C. A number of persons who have been living in a definite territory for a very long time with their own culture, tradition and government.

D. A society permanently living in a definite territory with a central authority, and executive responsible to the central authority and an independent judiciary.

Answer: A

As used in political science, the word state means a community or society politically organized under one independent government within a definite territory and subject to no outside control.

Source: https://www.iilsindia.com/study-material/528569_1600581365.pdf

Q68: With reference to Indian judiciary, consider the following statements:

1. Any retired judge of the Supreme Court of India can be called back to sit and act as a Supreme Court judge by the Chief Justice of India with prior permission of the President of India.

2. A High Court in India has the power to review its own judgement as the Supreme Court does.

Which of the statements given above is/are correct?

A. 1 only

B. 2 only

C. Both 1 and 2

D. Neither 1 nor 2

Answer: C

Explanation: Being a Court of record, the High Court is vested with powers to proceed under Article 226 of the Constitution of India itself and review a judgment,

Q69: With reference to India, consider the following statements:

1. There is only one citizenship and one domicile.

2. A citizen by birth only can become the Head of State.

3. A foreigner once granted citizenship cannot be deprived of it under any circumstances.

Which of the statements given above is/are correct?

A. 1 only

B. 2 only

C. 1 and 3

D. 2 and 3

Answer: A

Explanation: Citizen by Naturalization can also become Head of the state in India but not in USA

Deprivation is a compulsory termination of citizenship of India. A citizen of India by naturalization, registration, domicile and residence, may be deprived of his citizenship by an order of the Central Government if it is satisfied that:

The citizen has obtained citizenship by means of fraud, false representation or concealment of any material fact;

The citizen has shown disloyalty to the Constitution of India;

The citizen has unlawfully traded or communicated with the enemy during a war;

The citizen has, within five years after registration or neutralization, been imprisoned in any country for two years;

The citizen has been ordinarily resident out of India for seven years continuously.

Q70: Which one of the following factors constitutes the best safeguard of liberty in a liberal democracy?

A. A committed judiciary

B. Centralization of powers

C. Elected government

D. Separation of powers

Ans: D

Explanation: Centralization of powers gives rise to monarchy. Therefore decentralization of power is required for the safeguard of liberty. Powers should be divided into central, provincial and local self-governments. This decentralization of powers leads to efficient administration. And people also start cooperating with the functioning of the government.

Q71:  The vegetation of savannah consists of grassland with scattered small trees, but extensive areas have no trees. The forest development in such areas is generally kept in check by one or more or a combination of some conditions.

Which of the following are such conditions?

1. Burrowing animals and termites

2. Fire

3. Grazing herbivores

4. Seasonal rainfall

5. Soil properties

Select the correct answer using the code given below.

A. 1 and 2

B. 4 and 5

C. 2, 3 and 4

D. 1, 3 and 5

Answer: Debatable

Q72: With reference to the water on the planet Earth, consider the following statements:

1. The amount of water in the rivers and lakes is more than the amount of groundwater.

2. The amount of water in polar ice caps and glaciers is more than the amount of groundwater.

Which of the statements given above is/are correct?

A. 1 only

B. 2 only

C. Both 1 and 2

D. Neither 1 nor 2

Answer: B

Explanation:

Distribution of water on Planet Earth:

Streams and Rivers > Atmosphere > Soil Moisture > Lakes > Groundwater > Ice Caps and Glaciers > Oceans

Q73: Consider the following statements:

1. Moringa (drumstick tree) is a leguminous evergreen tree.

2. Tamarind tree is endemic to South Asia.

3. In India, most of the tamarind is collected as minor forest produce.

4. India exports tamarind and seeds of moringa.

5. Seeds of moringa and tamarind can be used in the production of biofuels.

Which of the statements given above are correct?

A. 1, 2, 4 and 5

B. 3, 4 and 5

C. 1, 3 and 4

D. 1, 2, 3 and 5

Ans: B

Statement 1 is incorrect: Moringa (botanical name Moringa oleifera) is a fast-growing, drought-resistant tree of the family Moringaceae, native to the Indian subcontinent. 

Statement 3 is correct: The Forest Rights Act confers ownership rights on minor forest produce like tamarind on tribal populations and residents of the particular forests. Today, about 90 per cent of the tamarind produced in the country comes from the forests.

Statement 4: India is the largest producer of moringa. So we can assume the option is correct.

Statement 5 is correct

Source: https://www.downtoearth.org.in/coverage/agriculture/sweet-n-sour-55668

Q74. The black cotton soil of India has been formed due to the weathering of

A. brown forest soil

B. fissure volcanic rock

C. granite and schist

D. shale and limestone

Answer: B

The Black soil are so called because of their black colouration and derived from the Basalt rock under semi-arid conditions. It is also known as ‘Regur’ (from the Telugu word Reguda) or black cotton soil as it is best suited for cotton cultivation. In India black soil are largely found over Deccan trap region of the states of Maharshtra, Madhya Pradesh, parts of Andhra Pradesh, Northern part of Karnataka, Gujarat, parts of Tamil Nadu and Rajasthan. Several theories have been put forward regarding the origin of this group of soils but most pedologists believe that these soils have been formed due to the solidification of lava spread over large areas during volcanic activity in the Deccan Plateau, in the Triassic Period. 

Q75. With reference to recent developments regarding ‘Recombinant Vector Vaccines’, consider the following statements:

1. Genetic engineering is applied in the development of three vaccines.

2. Bacteria and viruses are used as vectors.

Which of the statements given above is/are correct?

A. 1 only

B. 2 only

C. Both 1 and 2

D. Neither 1 nor 2

Answer: C

The use of recombinant proteins allows the targeting of immune responses focused against few protective antigens. There are a variety of expression systems with different advantages, allowing the production of large quantities of proteins depending on the required characteristics. Live recombinant bacteria or viral vectors effectively stimulate the immune system as in natural infections and have intrinsic adjuvant properties. DNA vaccines, which consist of non-replicating plasmids, can induce strong long-term cellular immune responses.

Q76. In the context of hereditary disease, consider the following statements:

1. Passing on mitochondrial diseases from parent to child can be prevented by mitochondrial replacement therapy either before or after in vitro fertilization of the egg.

2. A child inherits mitochondrial diseases entirely from mother and not from father.

Which of the statements given above is/are correct?

A. 1 only

B. 2 only

C. Both 1 and 2

D. Neither 1 nor 2

Answer: C

Both Statement 1 and 2 are correct.

Mitochondrial replacement therapy(MRT), sometimes called mitochondrial donation, is the replacement of mitochondria in one or more cells to prevent or ameliorate disease. 

MRT originated as a special form of in vitro fertilisation in which some or all of the future baby’s mitochondrial DNA (mtDNA) comes from a third party. This technique is used in cases when mothers carry genes for mitochondrial diseases. 

In Pronuclear transfer technique MRT is done after In vitro fertilisation (IVF).  Hence Statement 1 is true. 

A child inherits mitochondrial diseases only from Mother. Hence,  Statement 2 is true. 

Q77. Bollgard I and Bollgard II technologies are mentioned in the context of

A. Clonal propagation of crop plants

B. Developing genetically modified crop plants

C. Production of plant growth substances

D. Production of biofertilizers

Answer:  B.

Bollgard-1 and 2 are GM crops. 

Bollgard Bt cotton (single-gene technology) is India’s first biotech crop technology approved for commercialization in India in 2002, followed by Bollgard II- a double-gene technology in mid-2006, by the Genetic Engineering Approval Committee (GEAC) in India. 

Q78. In a pressure cooker, the temperature at which the food is cooked depends mainly upon which of the following?

1. Area of the hole in the lid

2. Temperature of the flame

3. Weight of the lid

Select the correct answer using the code given below. 

A. 1 and 2 only

B. 2 and 3 only

C. 1 and 3 only

D. 1,2 and 3 

Answer:  C.

In pressure cooker Technology, the temperature at which food is cooked is determined by the pressure inside it as volume is constant and pressure is regulated by the area of hole in the lid and weight of the lid.  The temperature of the flame doesn’t affect the temperature at which rice will be cooked. Hence, Statements 1 and 3 are true, while statement-2 is not true.  

Q79. Consider the following:

1. Bacteria

2. Fungi

3. Virus

Which of the above can be cultured in an artificial/synthetic medium?

A. 1 and 2 only

B. 2 and 3 only

C. 1 and 3 only

D. 1,2 and 3

Answer: A.   

Viruses replicate only within living cells be it bacteria, fungus or other living organisms. However, bacteria and fungus can be cultured in artificial mediums in laboratory conditions. 

Q80. . Consider the following statements :

1. Adenoviruses have single- stranded DNA genomes whereas retroviruses have double-stranded DNA genomes.

2. Common cold is sometimes caused by an adenovirus whereas AIDS is caused by a retrovirus.

Which of the statements given above is/are correct?

A. 1 only

B. 2 only

C. Both 1 and 2

D. Neither 1 nor 2

Answer:  B

The genome of adenovirus is double-stranded DNA, while the genome of a retrovirus is single-stranded RNA. Both types of viruses have been news over Corona Virus and development of Vaccines.  Hence, Statement-1 is not true. 

Common cold is caused by an adenovirus and other types of viruses, while AIDS is caused by the human immunodeficiency virus (HIV) – a subgroup of retrovirus. Hence, Statement-2 is true. 

Q81.  How is Permaculture farming different from conventional chemical farming?

Permaculture farming discourages monocultural practices but in conventional chemical farming, monoculture practices are predominant. 

Conventional chemical farming can cause increase in soul salinity but the occurrence of such phenomenon is not observed in permaculture farming

Conventional chemical farming is easily possible in semi-arid regions, but permaculture is not so easy in such regions. 

Practice of mulching is very important in permaculture farming, but not necessarily so in Conventional chemical farming.

Select the correct answer using the codes given below

A. 1 and 3 

B. 1,2 and 4

C. 4 only

D. 2 and 3 

Answer:   B

Since the principles of permaculture discourage monoculture, it opens up the opportunity for growing a wide variety of grains, fruits and vegetables, and widens one’s food basket.  It can be possible in all kinds of climate including semi- arid regions.  Hence, Statement-3 is not correct.

Permaculture cultivation discourages monoculture practices, Improves soil fertility and Encourages mulching.  Hence, Statements 1, 2 and 4 are true. 

https://www.downtoearth.org.in/coverage/environment/new-civilisation-primitive-wisdom-59125

Q82.  With reference to ‘palm oil, consider the following statements:

1. The palm oil tree is native to SouthEast Asia.

2. Palm oil is a raw material for some industries producing lipstick and perfumes.

3. The palm oil can be used to produce biodiesel 

Which of the statements given above are correct?

A. 1 and 2 only

B. 2 and 3 only

C. 1 and 3 only

D. 1, 2 and 3 only

Answer:   B

Palm oil is native to Africa and brought to southeast Asia during colonial times, palm oil plantations now cover huge areas of Malaysia and Indonesia.  

Palm oil is used in lipstick as it holds color well, doesn’t melt at high temperatures, and has a smooth application and virtually no taste.

Palm can be used for making biodiesel also. 

Hence, Statement-1 is not true while Statement-2 and 3 are true.

Q83. With reference to the Indus River system, of the following four rivers, three of them pour into one of them which joins the Indus directly. Among the following, Which one is such a river that joins the Indus directly?

A. Chenab

B. Jhelum

C. Ravi

D. Sutlej  

Answer:  D

About 17 kilometres north of Uch Sharif in Pakistan, the Sutlej unites with the Chenab river which already united with Jhelum and Ravi river, forming the Panjnad River. It finally flows into the Indus river about 100 kilometres west of the city of Bahawalpur in Pakistan. 

Q84.  With reference to India, Didwana, Kuchaman, Sargol and Khatu are the names of

A. glaciers

B. mangrove areas

C. Ramsar sites

D. saline lakes

Answer: D

All of them are saline lakes. 

Q85.  Consider the following rivers: 

  1. Brahmani
  2. Nagavali 
  3. Subarnarekha 
  4. Vamshandhra

Which of the above rise from Eastern Ghats? 

A. 1 and 2 

B. 2 and 4

C. 3 and 4 

D. 1 and 3

Answer:   B

Statement-1 and 3 are incorrect, while 2 and 4 are correct. 

Brahmani and Subarnarekha originate from central Indian Plateau regions. Vamsadhara River is an east-flowing river which originates in Kalahandi district of Odisha ie. Eastern Ghats.  The River Nagavali also known as Langulya is one of the main rivers of Southern Odisha and Northern Andhra Pradesh States in India, between Rushikulya and Godavari basins. 

Q86.  Consider the following statements: 

  1. The Global Ocean Commission grants licences for seabed exploration and mining in international waters. 
  2. India has received licences for seabed mineral exploration in international waters.
  3. ‘Rare earth minerals’ are present on the seafloor in international waters. 

Which of the statements given above are correct?

A. 1 and 2 only

B. 2 and 3 only

C. 1 and 3 only

D. 1, 2 and 3 ony

Answer: B

International Seabed Authority (ISA) set up under UNCLOS focuses mainly on organising, regulating and controlling all the mineral-related activities in the international seabed area that are beyond the limits of national jurisdiction. Hence, Statement-1 is not correct. 

India was the first country to receive the status of a ‘Pioneer Investor ‘ in 1987 and was given an area of about 1.5 lakh sq km in the Central Indian Ocean Basin (CIOB) for nodule exploration. In 2002, India signed a contract with the ISA and after complete resource analysis of the seabed 50% was surrendered and the country retained an area of 75,000 sq km. Ocean floors are rich sources of Rare Earth Minerals along with heavy metals.  

Hence, Statement-2 and 3 are correct. 

Q87. Among the following, which one of the least water- efficient crops? 

A. Sugarcane

B. Sunflower

C. Pearl millet

D. Red gram  

Answer: A .

Sugarcane is the least water efficient crop as evident from Droughts in Marathwada a few years ago. 

Q88. Consider the following Statements: 

  1. In the tropical zone, the western sections of the oceans are warmer than the eastern sections owing to the influence of the trade winds. 
  2. In the temperate zone, westerlies make the eastern sections of oceans warmer than the western sections. 

Which of the statements given above is/are correct?

A. 1 only

B. 2 only

C. Both 1 and 2 only

D. Neither 1 nor 2

Answer:  C. 

Normally, sea surface temperature is Western parts of the ocean rather than the eastern part. This is due to the trade winds blowing from east to west  carry warm surface water from east to west and allow upwelling of cold water in eastern coastal areas. Hence, Statement-1 is true. 

In the temperate zone,  Westerlies flowing from west to east bring warm water in eastern temperate regions; hence, temperature is slightly higher than eastern part. Hence, Statement-2 is true. 

Q89.  In the context of India’s preparation for Climate- Smart Agriculture, Consider the following statements: 

1.  The ‘ Climate- Smart Village’ approach in India is a part of a project led by the Climate Change, Agriculture and Food Security (CCAFS), an international research programme. 

2. The project CCAFS is carried out under Consultative Group on International Agricultural Research(CGIAR) headquartered in France. 

3. The international Crops Research Institute for the Semi- Arid Tropics (ICRISAT) in India is one of the CGIAR’s research centres. 

Which of the statements given above are correct?

A. 1 and 2 only

B. 2 and 3 only

C. 1 and 3 only

D. 1,2 and 3 

Answer: D

CCAFS project’s major activities in India include test, evaluate and develop portfolios of climate-smart interventions for different agro-ecological zones and farm types; promotion of Climate Smart Agriculture (CSAT) through the Climate-Smart Village (CSV) approach; weather-based insurance; use of ICT for dissemination of climate information based agro-advisories. 

CCAFS is carried out under CGIAR headquartered In France and ICRISAT located in Patancheru, Telangana is one of the CGIAR’s research centres. Hence, All the statements are true. 

Q90.  “ Leaf litter decomposes faster than in any other biome and as a result the soil surface is often almost bare. Apart from trees, the vegetation is largely composed of plant forms that reach up into the canopy vicariously, by climbing the trees or growing as epiphytes, rooted on the upper branches of trees”. This is the most likely description of 

A. Coniferous forest

B. Dry deciduous forests

C. Mangore forest

D. Tropical  rainforest

Answer:  D

All the characteristics mentioned in the question are of tropical rainforest. 

The hot and humid conditions lead to faster decay and absorption of nutrients.  Trees are only 70% of plants, the rest are creepers and climbers that reach up into the canopy and compete for water and sunlight. 

Q91. Water can dissolve more substances than any other liquid because:

A. It is Polar in nature

B. It is a good conductor of heat

C. It has a high value of specific heat

D. It is an oxide of hydrogen

Answer – A (polarity of water)

Source:- https://www.usgs.gov/special-topic/water-science-school/science/water-qa-why-water-universal-solvent?qt-science_center_objects=0#qt-science_center_objects

Q92. With Reference to street lighting, how are sodium lamps different from LED Lamps?

  1. Sodium lamp produces light in 360 degrees but it is not so in the case of LED lamps
  2. As Street lights, sodium lamps have a longer lifespan than street lamps.
  3. The spectrum of visible light from sodium lamps is almost monochromatic while LED lamps offer significant colour advantages in street lightning

Select the correct answer from the code given below

A. 3 only

B. 2 only

C. 1 and 3 only

D. 1, 2 and 3 only

Answer:  C

What’s the Major Upside to LED Lights?

There are four major advantages to LED lighting:

LEDs have an extremely long lifespan relative to every other lighting technology. 

New LEDs can last 50,000 to 100,000 hours or more. The typical lifespan for a fluorescent bulb, by comparison, is 10-25% as long at best (roughly 10,000 hours).

LEDs are extremely energy efficient relative to every other commercially available lighting technology. There are several reasons for this to include the fact they waste very little energy in the form of infrared radiation (much different than most conventional lights to include fluorescent lights), and they emit light directionally (over 180 degrees versus 360 degrees which means there are far fewer losses from the need to redirect or reflect light).

Very high light quality.

Very low maintenance costs and hassle

Sodium vapour lights have the worst colour rendering on the market. LPS lamps in particular are monochromatic which means that the objects illuminated by them appear shadowy black rather than the colour you would see during daylight. HPS lamps are better but are still surpassed by virtually every other light on the market.  

https://www.stouchlighting.com/blog/led-vs-hps-lps-high-and-low-pressure-sodium

https://www.energy.gov/energysaver/led-lighting

Q93. The term ACE-2 is talked about in the context of?

A. Genes introduced in the genetically modified crops

B. Development of India’s own navigation satellite system

C. Radio collar for wildlife trafficking

D. Spread of viral diseases

Answer – D

Solution:- 

The SARS-CoV-2 virus enters a human cell through ‘ACE2 receptors’, which are enzymes that generate small proteins that are useful for the cell. The coronaviruses have horn-like projections, called spike proteins. The spike proteins bind to ACE2, like a key inserted into a lock, and the virus gains entry into the cell and infects it. 

Source The Hindu https://www.google.com/amp/s/www.thehindubusinessline.com/business-tech/ace2-receptors-how-the-sars-cov-2-virus-transmits-so-fast/article33956645.ece/amp/

Q94. Bisphenol A(BPA), a cause of concern is a structural/key component in the manufacture of which of the following kinds of plastic?

A. Low-density Polyethylene

B. Polycarbonate

C. Polyethylene terephthalate

D. polyvinyl chloride

Answer- B

Solution:-  In the news because BPA mimics estrogen and was found in feeding bottles of infants. It is considered carcinogenic and linked with lower IQ in newborns

Source:- although it is an old news, yet every time you look at any water bottle it says BPA free.

https://www.google.com/amp/s/indianexpress.com/article/india/study-shows-use-of-banned-synthetic-in-feeding-bottles-5741631/lite/

Q95. Triclosan is considered harmful when exposed to. high levels for a very long time, is most likely present in which of the following?

A. food preservatives

B. Food ripening substance

C. Reused plastic containers 

D. Toiletries

Solution:- D 

In news due to Covid hand washing protocols. A study by IIT HYDERABAD found its harmful uses. Common sense discussion – the killing of good bacteria due to frequent hand washing

Source:- 

https://m.timesofindia.com/city/hyderabad/triclosan-in-soap-toothpaste-harmful-to-health-iit-h/amp_articleshow/79747418.cms

Q96. Which one of the following is the reason why astronomical distances are measured in light years?

A. Distance among stellar bodies do not change

B. Gravity of stellar bodies do not change

C. Light always travels in a straight line

D. Speed of light is always the same

Answer: D

Solution:- The only thing absolute in this universe according to Einstein is the speed of light, rest everything is relative. So to get an idea of interstellar distance, Light Year is the best method, as it is really large and appropriate for such lengths and is even concise, as no one would remember all the digits or exponents when the distance is expressed in Kilometres.

Q97. We adopted our parliamentary democracy based on the British model,  but how does our model differ from the British model?

  1. As Regards legislation the British parliament is Supreme or sovereign, but in India the power of the parliament to legislate is limited.
  2. In India matters related to the constitutionality of an amendment of an act of the parliament are referred to the constitutional bench by the Supreme Court

Select the correct answer using the code given below:

A. 1 only

B. 2 only 

C. Both 1 and 2

D. Neither 1 nor 2

Answer: C

Solution :- Laxmikant chapter Parliamentary System last page – Distinction between indian and British Models

Q98. With reference to the union government consider the following statement

  1. N Gopalaswamy Ayengar Committee suggested that a minister and a Secretary be designated solely for pursuing the subject of administrative reform and promoting it.
  2. In 1970 the Department of Personnel was constituted on the recommendation of the Administrative Reforms Commission 1966 and this was placed under the Prime Minister’s charge.

Which of the following statements given above is/are correct?

A. 1 only

B. 2 only

C. Both 1 and 2

D. Neither 1 nor 2

Answer: B

Source :- https://darpg.gov.in/about-department-0

Q99. Right to Privacy is protected under which article of the constitution of India?

A. Art 15

B. Art 19

C. Art 21

D. Art 29

Solution :- C

Direct and repeated question. Justice K S Puttaswamy judgement of supreme court asked before in main exam as well

Source:- https://www.civilsdaily.com/news/surveillance-and-human-right/

Q100. Consider the following statement

  1. In India there is no law restricting the candidate from contesting in one Lok Sabha election from 3 constituencies
  2. In 1991 lok sabha  elections Sri Devi Lal contested from 3 lok sabha constituencies
  3. As per the existing rules if a  candidate contests  in one Lok Sabha elections from many constituencies, his/her party  should bear the cost of bye elections vacated by him or her in the event of him/her winning in all the constituencies

Which of the statements given above is/ are correct? 

A. 1 only

B. 2 only

C. 1 and 3 only

D. 2 and 3 only

Answer- A 

Solution :- Laxmikant Electoral Reform page 71.3 contestants restricted to two constituencies

Bhavabhuti was an 8th-century scholar of India noted for his plays and poetry, written in Sanskrit. His plays are considered equal to the works of Kalidasa.

Hastimalla wrote 8 plays including VikrantKaurava and Subhadra Harana. He was a noted Kannada poet and playwright in the Hoysala Empire.

Q2. Consider the following statements: (Tikdam)

1. The Montagu Chelmsford Reforms of 1919 recommended granting voting rights to all women above the age of 21.

2. The Government of India Act of 1935 gave women reserved seats in the legislature.

Which of the statements given above is/are correct?

A. 1 only

B. 2 only

C. Both 1 and 2 

D. Neither 1 nor 2

Answer: B

The 1st statement is wrong. When the Montagu–Chelmsford Reforms were introduced in 1918 no recommendation was made for Indian women’s enfranchisement. Suffragists were active in drawing up petitions and published updates about the struggle in Stri Dharma, urging support for women’s political empowerment as a part of the anti-colonial movement against Britain. The Act did not grant women suffrage but included a clause that Indian provinces could enfranchise women if they chose to do so. It limited suffrage, barring most of India’s middle class, as it restricted the vote to those who had an annual income of more than ₹10,000–20,000; land revenues in excess of ₹250–500 per annum; or those recognised for their high level of public work or scholarship. Furthermore, it did not allow women to stand in elections. The law empowered the Imperial Legislative Assembly and the Council of State to grant the right to vote in those provinces in which legislative franchise had been approved, but the British Parliament retained the right to determine who could stand as candidates for the Legislative Councils.

The second statement is correct. The Government of India Act, 1935, as finally adopted, made twenty-nine million men and six million women eligible to vote. Seats were reserved for women on a communal basis while women could contest from any general seat as well.

Q3. With reference to 8th August 1942 in Indian History, which one of the following statements is correct?

A. The Quit India Resolution was adopted by the AICC.

B. The Viceroy’s Executive Council was expanded to include more Indians

C. The Congress ministries resigned in seven provinces.

D. Cripps proposed an Indian Union with full Dominion Status once the Second World War was over.

Answer: A

It’s an easy question. On August 8, 1942, the Quit India Resolution was passed at the Bombay session of the All India Congress Committee (AICC). On the same day, Gandhi urged Indians to act as a free nation and follow non-violent civil disobedience. This took place in August Kranti Maidan.

On the other hand, on 8 August 1940, the Viceroy Lord Linlithgow made a proposal called the August Offer which expanded the Executive Council to include more Indians. 

The Congress ministries resigned in October and November 1939, in protest against Viceroy Lord Linlithgow’s action of declaring India to be belligerent in the Second World War without consulting the Indian people.

The Cripps Mission happened in the months of March and April of 1942.

Q4. Who among the following is associated with ‘Songs from Prison’, a translation of Ancient Indian religious lyrics in English?

A Bal Gangadhar Tilak

B. Jawaharlal Nehru

C Mohandas Karamchand Gandhi

D. Sarojini Naidu

Answer: C

‘Songs from Prison’ was a translation from Sanskrit hymns and lyrics, from the Upanishads and other scriptures. Gandhi made these translations during his incarceration in Yeravda Prison in 1930. The other editor was John S Hoyland.

Q5. With reference to Medieval India, which one of the following is the correct sequence in ascending order in terms of size?

A. Pargana-Sarkar-Suba

B. Sarkar-Paragana-Suba

C. Suba-Sarkar-Pargana

D. Pargana-Suba-Sarkar

Answer: A

The Mughal Empire was essentially military in nature where the word of the emperor was law. The administrative structure was highly centralized as viewed by historians like Irfan Habib, Athar Ali, etc. Mughal Empire was divided into Suba; Suba was further divided into Sarkar, and Sarkar was divided further into Pargana and Pargana had various Villages under them. 

Q6. Who among the following was associated as Secretary with Hindu Female School which later came to be known as Bethune Female School?

A. Annie Besant

B. Debendra Nath Tagore

C. Ishwar Chandra Vidyasagar

D. Sarojini Naidu

Answer: C

This year, on September 4th, this institution was in the news when a bust of Ishwar Chandra Vidyasagar was unveiled in the institute. The college owes its origin to John Elliot Drinkwater Bethune. What began as Hindu Female School in 1849 was renamed Bethune School in 1856. In 1856, the Government took charge of the Hindu Female School, later renamed Bethune School. The Managing Committee of the school was then formed and Pandit Ishwar Chandra Vidyasagar was made the Secretary. In August 1878, Bethune School was amalgamated with Banga Maha Vidyalaya which was founded by Miss Annette Akroyd with the help of Durgamohan Das, Dwarka Nath Ganguly and Anandamohan Basu

Q7. In the context of Colonial India, Shah Nawaz Khan, Prem Kumar Sehgal and Gurbaksh Singh Dhillon are remembered as

A. Leaders of Swadeshi and Boycott Movement

B. Members of the Interim Government in 1946.

C. Members of the Drafting Committee in the Constituent Assembly.

D. Officers of the Indian National Army.

Answer: D

Quite an easy question once again. You must have read about the famous Red Fort Trials. Major General Shah Nawaz Khan, Colonel Gurbaksh Singh Dhillon and Colonel Prem Kumar Sahgal were three of the senior-most officers of INA and trusted colleagues of Netaji. They underwent court-martial by the British at Red Fort in 1945 and were sentenced to death, but instead had to be released following widespread protests and unrest in India. Congress leader and the country’s first Prime Minister, Jawaharlal Nehru, also got on board the INA officers’ legal defence team, along with party colleague Bhulabhai Desai and barrister Tej Bahadur Sapru.

Q8.  With reference to Indian history, which of the following statements is/are correct?

  1. The Nizamat of Arcot emerged out of Hyderabad state.
  2. The Mysore Kingdom emerged out of Vijayanagara Empire.
  3. The Rohilkhand Kingdom was formed out of the territories occupied by Ahmad Shah Durrani.

Select the correct answer using the code given below:

A. 1 and 2 only

B. 2 only

C. 2 and 3 only

D. 3 only

Answer: A

The Nawabs of the Carnatic (also referred to as the Nawabs of Arcot) were the nawabs who ruled the Carnatic region of South India between about 1690 and 1855. The Carnatic was a dependency of Hyderabad Deccan, and was under the legal purview of the Nizam of Hyderabad, until their demise. 

The second statement is correct. The Mysore kingdom, which was founded and ruled for the most part by the Hindu Wodeyar family, initially served as a vassal state of the Vijayanagara Empire. After the decline of the Vijayanagar Empire, Mysore became an independent state under the Hindu Wodeyar Dynasty in AD 1565. 

Regarding the third statement, it is incorrect because Nawab Ali Mohammed Khan, a scion of the ancient Barha Dynasty became the first Nawab of Rohilkhand, having been previously elected as overlord by various Afghan Chiefs at the age of fourteen. He would carve out the future kingdom from the collapsing Mughal Empire and go on to found the Rohilla Dynasty. The crown would go on being held by the Rohillas until the kingdom came to an end in 1774, and thereafter the same dynasty would rule over Rampur. Most of Rohilkhand’s borders were established by Ali Mohammed Khan and largely came into existence as a check to the power of the Oudh State.

Ahmad Shah Durrani was the founder of the Durrani Empire and is regarded as the founder of the modern state of Afghanistan. Within a few years, he extended his control from Khorasan in the west to Kashmir and North India in the east, and from the Amu Darya in the north to the Arabian Sea in the south.

Q9. Which one of the following statements is correct?

A. Ajanta Caves lie in the gorge of the Waghora river.

B. Sanchi Stupa lies in the gorge of the Chambal river.

C. Pandu-lena Cave Shrines lie in the gorge of the Narmada River.

D. Amravati Stupa lies in the gorge of the Godavari river.

Answer: A

The first statement is correct. The Ajanta caves are cut into the side of a cliff that is on the south side of a U-shaped gorge on the small river Waghora. 

The second statement is wrong. Sanchi Stupa lies in an upland plateau region, on a flat-topped sandstone hill, just west of the Betwa River and about 5 miles (8 km) southwest of Vidisha.

The third statement is wrong. The Buddhist monuments Pandavleni Caves also known by the names Pandu Lena caves and Trirashmi caves are a group of 24 rock-cut caves. It is located high up on the north face of Trivashmi hill 8 km south of Nasik city. The donative inscriptions indicate that these excavations are belonging to the Satavahana and Kshatrapa periods, spanning 2 C BC to the 3C AD. Buddha images were added to many of the caves in 5C-6C. The nearest river is the Godavari.

The fourth statement is wrong. Amravati stupa was founded by an emissary of Emperor Ashoka and its construction got completed around 2000 years ago. Amravati Stupa illustrates Lord Buddha in a human form and He is shown subduing an elephant. It is taller than the Sanchi stupa and has 95 ft high platforms, extending in the four cardinal directions, along with a huge circular dome. It is situated near river Krishna.

Q10. Consider the following statements:

  1. 21st February is declared to be International Mother Language Day by UNICEF.
  2. The demand that Bangla has to be one of the national languages was raised in the Constituent Assembly of Pakistan.

Which of the above statements is/are correct?

A. 1 only

B. 2 only

C. Both 1 and 2

D. Neither 1 nor 2

Answer: C

The first statement is incorrect because of the UNESCO/UNICEF angle which we will come to later. The Constituent Assembly of Pakistan was in session at Karachi-then the capital of Pakistan-from 23 February 1948. It was proposed that the members would have to speak either in Urdu or in English at the Assembly. Dhirendra Nath Datta, a member of the East Pakistan Congress Party, moved an amendment motion to include Bangla as one of the languages of the Constituent Assembly.  In the same year, the Government of the Dominion of Pakistan ordained Urdu as the sole national language, sparking extensive protests among the Bengali-speaking majority of East Bengal. Facing rising sectarian tensions and mass discontent with the new law, the government outlawed public meetings and rallies. The students of the University of Dhaka and other political activists defied the law and organised a protest on 21 February 1952. The movement reached its climax when police killed student demonstrators on that day. The deaths provoked widespread civil unrest. After years of conflict, the central government relented and granted official status to the Bengali language in 1956. In Bangladesh, 21 February (Ekushey February) is observed as Language Movement Day, a national holiday. In 1999, UNESCO declared 21 February as International Mother Language Day, in tribute to the Language Movement and the ethnolinguistic rights of people around the world.

Q11. With reference to Chausath Yogini Temple situated near Morena, consider the following statements:

  1. It is a circular temple built during the reign of the Kachchhapaghata Dynasty.
  2. It is the only circular temple built in India.
  3. It was meant to promote the Vaishnava cult in the region.
  4. Its design has given rise to a popular belief that it was the inspiration behind the Indian Parliament building.

Which of the statements given above are correct?

A. 1 and 2 only

B. 2 and 3 only

C. 1 and 4 only

D. 2, 3 and 4 only

Answer: C

Tikdam is easily applicable here. With a little bit of knowledge, you know that statement 4 is correct. On the other hand, the 2nd statement is too extreme. That gives us the answer which is 1 and 4 only.

According to an inscription, the temple was constructed by Maharaja Devapala of the Kacchapagata dynasty, dated VS 1380 (1323 CE). This temple is situated on top of a small hill and shows a circular plan. The temple is built on a high plinth and shows pillared cloisters that run around the wall facing an open courtyard. The small cells that form 64 subsidiary shrines have a shallow pillared mandapa in front; while a circular main shrine facing east stands in the middle of the courtyard. The cells and the main shrine are flat-topped, but it is believed that initially, each had a shikhara on top. While the 64 Yoginis originally placed in the 64 subsidiary shrines are now missing, a Shiva linga has taken their places in each cell. 

Q12. Which one of the following ancient towns is well-known for its elaborate system of water harvesting and management by building a series of dams and channelizing water into connected reservoirs?

A. Dholavira

B. Kalibangan

C. Rakhigarhi

D. Ropar

Answer: A

The site was in news quite recently and it is expected that you know this. Dholavira is located on Kadir island in the Rann of Kutch in Gujarat. The city had an impressive and unique water harvesting and management system. It can be noted that this area receives less than 160 cm of rain every year and is very prone to droughts.  A sophisticated water management system demonstrates the ingenuity of the Dholavira people in their struggle to survive and thrive in a harsh environment.

Q13. In the first quarter of the seventeenth century, in which of the following was/were the factory/factories of the English East India Company located?

  1. Broach
  2. Chicacole
  3. Trichinopoly

Select the correct answer using the code given below:

A. 1 only

B. 1 and 2 only

C. 3 only

D. 2 and 3 only

Answer: A

By 1623, the English East India Company had established factories at Surat, Broach, Ahmedabad, Agra, and Masulipatam. 

Q14. From the decline of Guptas until the rise of Harshavardhana in the early seventh century, which of the following kingdoms were holding power in Northern India?

  1. The Guptas of Magadha
  2. The Paramaras of Malwa
  3. The Pushyabhutis of Thanesar
  4. The Maukharis of Kanauj
  5. The Yadavas of Devagiri
  6. The Maitrakas of Valabhi

Select the correct answer using the code given below.

A. 1, 2 and 5

B. 1, 3, 4 and 6

C. 2, 3 and 4

D. 5 and 6

Answer: B

The Later Gupta dynasty ruled the Magadha region in eastern India between the 6th and 7th centuries CE. The Later Guptas succeeded the imperial Guptas as the rulers of Magadha. So 1st is correct. The Paramaras of Malwa rose on the ruins of the Pratihara empire of Kanauj in 11 the Century. So statement 2 is incorrect.  The Maukharis (554 CE – 606 CE) rose as a power after the downfall of the Gupta Empire (3rd to 6th century CE) in the 6th century CE in northern India.  So 3rd is correct.

Q15. According to the Portuguese writer Nuniz, the women in the Vijayanagara Empire were expert in which of the following areas?

1. Wrestling

2. Astrology

3. Accounting

4. Soothsaying

Select the correct answer using the code given below.

A. 1, 2 and 3 only

B. 1, 3 and 4 only

C. 2 and 4 only

D. 1, 2, 3 and 4

Answer: D

All claims are correct.

Q16. With reference to Madanapalle of Andhra Pradesh, which one of the following statements is correct?

A. Pingali Venkayya designed the tricolour Indian National Flag here.

B. Pattabhi Sitaramaih led the Quit India Movement of Andhra Region from here.

C. Rabindranath Tagore translated the National Anthem from Bengali to English here.

D. Madame Blavatsky and Colonel Olcott set up the headquarters of Theosophical Society first here.

Answer: C 

Rabindranath Tagore translated “Jana Gana Mana” from Bengali to English and also set it to music in Madanapalle.

Q17. Consider the following pairs:

(Historical place) (Well-known for)

1. Burzahom : Rock-cut shrines

2. Chnadra-ketugarh : Terracotta art

3. Ganeshwar : Copper artefacts

Which of the pairs given above is/are correctly matched?

A. 1 only

B. 2 only

C. 3 only

D. 2 and 3

Answer: D

Chandraketugarh features many examples of terracotta art, displaying an unusual degree of precision and craftsmanship. These plaques are comparable to those found at other better-known sites such as Kaushambi and Ahichhatra. The terracotta plaques from these sites often carry similar motifs executed in nearly identical fashion. This similarity suggests an established communication link and common cultural heritage among these sites.

Ganeshwar had mainly supplied copper objects to Harappa. The archaeological site was named after the present-day village, Gilund, and is locally known as Modiya Magari which means “bald habitation mound”

Q18. Consider the following statements:

  1. It was during the reign of Iltutmish that Chengiz Khan reached the Indus in pursuit of the fugitive Khwarezm prince.
  2. It was during the reign of Muhammad bin Tughluq that Taimur occupied Multan and crossed the Indus.
  3. It was during the reign of Deva Raya 2 of Vijayanagara Empire that Vasco da Gama reached the coast of Kerala.

Which of the statements given above is/are correct?

A. 1 only

B 1 and 2

C. 3 only

D. 2 and 3

Answer: A

Q19. Consider the Following Statements:

1. St. Francis Xavier was one of the founding members of the Jesuit Order.

2. St. Francis Xavier died in Goa and a church is dedicated to him there.

3. The Feast of St. Francis Xavier is celebrated in Goa each year.

Which of the statements given above are correct?

A. 1 and 2 only

B. 2 and 3 only

C. 1 and 3 only

D. 1, 2 and 3

Answer: C

He died in China and hence once you eliminate this wrong statement using this Tikdam, you come to the right answer.

Q20. With reference to the history of ancient India, which of the following statements is/are correct?

1. Mitakshara was the civil law for upper castes and Dayabhaga was the civil law for lower castes.

2. In the Mitakshara system, the sons can claim right to the property during the lifetime of the father, whereas in the Dayabhaga system, it is only after the death of the father that the sons can claim right to the property.

3. The Mitakshara system deals with the matter related to the property held by male members only of a family, whereas the Dayabhaga system deals with the matters related to the property held by both male and female members of a family.

Select the correct answer using the code given below.

A. 1 and 2

B. 2 only

C. 1 and 3

D. 3 only

Answer: B

The Dāyabhāga is a Hindu law treatise written by Jīmūtavāhana which primarily focuses on inheritance procedure. The Dāyabhāga was the strongest authority in Modern British Indian courts in the Bengal region of India, although this has changed due to the passage of the Hindu Succession Act of 1956 and subsequent revisions to the act. Based on Jīmūtavāhana’s criticisms of the Mitākṣarā, it is thought that his work is precluded by the Mitākṣarā. This has led many scholars to conclude that the Mitākṣarā represents the orthodox doctrine of Hindu law, while the Dāyabhāga represents the reformed version

Q21. The money multiplier in an economy increases with which one of the following?

A. Increases in the Cash Reserve Ratio in the banks

B. Increases in the Statutory Liquidity Ratio in the banks

C. Increase in the banking habit of the people

D. Increases in the Population of the country

Answer: C

Q22. With reference to the Indian economy, demand-pull inflation can be caused/increased by which of the following?

1. Expansionary policies

2. fiscal stimulus

3. Inflation-indexing wages

4. Higher purchasing power

5. Rising interest rates

Select the correct answer using the code given below.

A. 1, 2 and 4 only

B. 3, 4 and 5 only

C 1, 2, 3 and 5 only

D. 1, 2, 3, 4 and 5

Answer: A

Q23. With reference to India, consider the following statements:

1. Retail investors through demat accounts can invest in ‘Treasury Bills’ and ‘Government of India Debt Bonds’ in the primary market.

2. The ‘Negotiated Dealing System Order Matching’ is a Government securities trading platform of the Reserve Bank of India.

3. The ‘Central Depository Services Ltd.’ Is jointly promoted by the Reserve Bank of India and the Bombay Stock Exchange.

Which of the statements given above is/are correct?

A. 1 only

B. 1 and 2

C. 3 only

D. 2 and 3

Answer: B

Promoters/Shareholders

CDSL is promoted by Bombay Stock Exchange Ltd (BSE Ltd) – Asia’s latest Stock Exchange. Other shareholders include HDFC Bank Ltd, Standard Chartered Bank and Canara Bank.

NDS-OM is a screen-based electronic anonymous order matching system for secondary market trading in Government securities owned by RBI. Presently the membership of the system is open to entities like Banks, Primary Dealers, Insurance Companies, Mutual Funds etc. i.e entities who maintain SGL accounts with RBI.

Q24. With reference to ‘WaterCredit’, consider the following statements:

1. It puts microfinance tools to work in the water and sanitation sector.

2. It is a global initiative launched under the aegis of the World Health Organization and the World Bank.

3. It aims to enable the poor people to meet their water needs without depending on subsidies.

Which of the statements given above are correct?

A. 1 and 2 only

B. 2 and 3 only

C. 1 and 3 only

D. 1, 2 and 3

Answer: C

The Water Credit Initiative has been funded by social entrepreneur Gary White and Hollywood Actor Matt Damon through their organization Water.org which has invested US$ 2.2 million in Water Credit programs. Water credit applies the principles of microfinance to the water and sanitation sector in developing countries. By making small loans to individuals and communities who do not have access to credit, Water-Credit empowers people to address their own water and sanitation needs instead of depending on government funds and charity.

Q25. In India, the central bank’s function as the ‘lender of last resort’ usually refers to which of the following?

1. Lending to trade and industry bodies when they fail to borrow from other sources

2. Providing liquidity to the banks having a temporary crisis

3. Lending to governments to finance budgetary deficits

Select the correct answer using the code given below:

A. 1 and 2

B. 2 only

C. 2 and 3

D. 3 only

Answer: B

Q26. ‘R2 code of practices’ constitutes a tool available for promoting the adoption of

A. environmentally responsible practices in electronics recycling industry

B. ecological management of ‘Wetlands of International importance’ under the Ramsar Convention

C. sustainable practices in the cultivation of agricultural crops in degraded lands

D. ‘Environmental Impact Assessment’ in the exploitation of natural resources

Answer: A

R2 means responsible recycling. R2 stands for Responsible Recycling and is a standard specifically created for the electronics recycling industry by Sustainable Electronics Recycling International (SERI). The standards were then accredited by ANAB, and in 2008, R2 was released.

Q27. Why there is a concern about copper smelting plants?

1. They may release lethal quantities of carbon monoxide into the environment.

2. The copper slag can cause the leaching of some heavy metals into the environment.

3. They may release sulphur dioxide as a pollutant.

Select the correct answer using the codes given below.

A. 1 and 2 only

B. 2 and 3 only

C. 1 and 3 only

D. 1, 2 and 3

Answer: B

The smelting of sulfide ores results in the emission of sulfur dioxide gas, which reacts chemically in the atmosphere to form a sulfuric acid mist.

Q28. With reference to furnace oil. Consider the following statements:

1. It is a product of oil refineries.

2. Some industries use it to generate power.

3. Its use causes sulphur emissions into the environment.

Which of the statements given above are correct?

A. 1 and 2 only

B. 2 and 3 only

C. 1 and 3 only

D. 1, 2 and 3

Answer: D

Q29. What is blue carbon?

A. Carbon captured by oceans and coastal ecosystems

B. Carbon sequestered in forest biomass and agricultural soils

C. Carbon contained in petroleum and natural gas

D. Carbon present in atmosphere

Answer: A

Q30. In the nature, which of the following is/are most likely to be found surviving on a surface without soil?

1. Fern

2. Lichen

3. Moss

4. Mushroom

Select the correct answer using the code given below.

A. 1 and 4 only

B. 2 only

C. 2 and 3

D. 1, 3 and 4

Answer: C

Q31. Consider the following statements:

1. The Governor of the Reserve Bank of India (RBI) is appointed by the Central Government.

2. Certain provisions in the Constitution of India give the Central Government the right to issue directions to the RBI in the public interest.

3. The Governor of the RBI draws his power from the RBI Act. 

Which of the above statements are correct?

A. 1 and 2 only

B. 2 and 3 only

C. 1 and 3 only.

D. 1, 2 and 3

Answer: C

The statement 1 is correct – the RBI Governor and Deputy Governors are appointed by the Central Government. Their names are cleared by the Cabinet Committee on appointments.

The statement 2 is wrong – According to Section 7 of the RBI Act the central government is empowered to issue directions it considers necessary for public interest to the central bank from time to time after consultation with the RBI governor. It is not the constitution.

Q32. With reference to casual workers employed in India, consider the following statements :

1. All casual workers are entitled to Employees Provident Fund coverage.

2. All casual workers are entitled for regular working hours and overtime payment.

3. The government can by a notification specify that an establishment or industry shall pay wages only through its bank account.

Which of the above statements are correct?

A. 1 and 2 only

B. 2 and 3 only 

C. 1 and 3 only

D. 1, 2 and 3

Answer: A 

Entitlements of the provident fund to be extended to contractual employees: Supreme Court

The Hon’ble Supreme Court of India, vide its judgment dated 17.01.2020 in the matter of M/s. Pawan Hans Limited & Ors. Vs. Aviation Karmachari Sanghatana & Ors. clarified that the contractual employees who are drawing wages/salary directly or indirectly are entitled to the Provident Fund Benefits.

The judgment by SC definitely brings a respite to the contractual employees of the Organization as well as other nationwide employees in such a situation, who had been seeking equality with the regular employees under the PF Trust Regulations of India.

https://www.greythr.com/blog/whether-contractual-employees-are-entitled-pf-benefits/

statement 1 is right

The Payment of Wages  Act, 1936 has been amended by Payment of Wages (Amendment) Act, 2017 (effective from 28.12.2016) to enable the employers to pay wages to their employees by (a) cash or (b) cheque or (c) crediting to their bank account. The amendment in the Act also enables the appropriate Government to specify the industrial or other establishment, by notification in the Official Gazette, which shall pay to every person employed in such industrial or other establishment the wages only by cheque or by crediting in his bank account.

https://pib.gov.in/newsite/PrintRelease.aspx?relid=187043

Statement 3 is wrong

Q33. Which among the following steps is most likely to be taken at the time of an economic recession?

A. Cut in tax rates accompanied by increase in interest rate 

B. Increase in expenditure on public projects

C. Increase in tax rates accompanied 

D. Reduction of expenditure on public projects

In case of Economic Recession, we need to increase the money supply into the economy.

Ways to boost money into the economy:

Cutting the tax rate

Reducing interest rate – easier available loans and credit

Increase the public expenditure

Answer – B

Q34. Consider the following statements: 

Other things remaining unchanged, market demand for a good might increase if

1. price of its substitute increases 

2. price of its complement increases

3. the good is an inferior good and the income of the consumers increases its price falls

4. Its prices fall

Which of the above statements are correct?

A. 1 and 4 only

B. 2, 3 and 4

C. 1, 3 and 4

D. 1, 2 and 3

Answer: C

When the price of a substitute for a good falls, the demand for that good will decline and when the price of the substitute rises, the demand for that good will increase.

The goods which are complementary with each other, the fall in the price of any of them would favorably affect the demand for the other.

The greater income means the greater purchasing power. Therefore, when incomes of the people increase, they can afford to buy more. It is because of this reason that increase in income has a positive effect on the demand for a good.

Q35. With reference to ‘Urban Cooperative Banks’ in India, consider the following statements : 

1. They are supervised and regulated by local boards set up by the State Governments. 

2. They can issue equity shares and preference shares. 

3. They were brought under the purview of the Banking Regulation Act, 1949 through an Amendment in 1966.

Which of the statements given above is/are correct?

A. 1 only

B. 2 and 3 only

C. 1 and 3 only

D. 1, 2 and 3

Answer: B 

Statement 1 is wrong

RBI is the one regulating them now.

Statement 2 – is correct

The Reserve Bank on Wednesday came out with draft guidelines allowing primary urban cooperative banks (UCBs) to augment capital through issuance of equity shares, preference shares and debt instruments.

https://www.business-standard.com/article/pti-stories/rbi-issues-draft-circular-on-allowing-ucbs-to-augment-capital-121071400994_1.html

Statement 3 is correct – They were brought under the RBI’s watch in 1966, a move that brought the problem of dual regulation along with it.

Q36. Indian Government Bond Yields are influenced by which of the following? 

1. Actions of the United States Federal Reserve 

2. Actions of the Reserve Bank of India 

3. Inflation and short-term interest rates

Select the correct answer using the code given below.

A. 1 and 2 only

B. 2 only

C. 3 only

D. 1, 2 and 3

Answer: D

The multifaceted roles played by the RBI in the payment system, monetary policy, financial stability policy, and policy coordination with the Treasury gives it the operational ability to influence government bonds’ nominal yields by setting and changing the short-term interest rate and using other tools of monetary policy as it deems appropriate.

Statement 2 – correct

https://www.adb.org/sites/default/files/publication/488961/adr-vol36no1-7-government-bond-yields-india.pdf

Statement 3 is also correct

Q37. Consider the following:

1. Foreign currency convertible bonds 

2. Foreign institutional investment with certain conditions 

3. Global depository receipts 

4. Non-resident external deposits

Which of the above can be included in Foreign Direct Investments?

A. 1, 2 and 3

B. 3 only

C. 2 and 4

D. 1 and 4

Answer: D

FIIs can include hedge funds, insurance companies, pension funds, investment banks, and mutual funds. FIIs can be important sources of capital in developing economies, yet many developing nations, such as India, have placed limits on the total value of assets an FII can purchase and the number of equity shares it can buy, particularly in a single company.3 This helps limit the influence of FIIs on individual companies and the nation’s financial markets, and the potential damage that might occur if FIIs fled en masse during a crisis.

https://www.investopedia.com/terms/f/fii.asp

Option 2 is wrong

A global depositary receipt (GDR) is a certificate issued by a bank that represents shares in a foreign stock on two or more global markets.

GDRs typically trade on American stock exchanges as well as Eurozone or Asian exchanges.

GDRs and their dividends are priced in the local currency of the exchanges where the shares are traded.

GDRs represent an easy, liquid way for U.S. and international investors to own foreign stocks.

https://www.investopedia.com/terms/g/gdr.asp

Option 3 is wrong

Q38. Consider the following statements:

The effect of devaluation of a currency is that it necessarily

1. Improves the competitiveness of the domestic exports in the foreign markets

2. increases the foreign value of domestic currency 

3. improves the trade balance 

Which of the above statements is/are correct?

A. 1 only

B. 1 and 2

C. 3 only

D. 2 and 3

https://www.economicshelp.org/macroeconomics/exchangerate/effects-devaluation/

A devaluation of the exchange rate will make exports more competitive and appear cheaper to foreigners. This will increase demand for exports. 

statement 1 is correct.

A devaluation in the Pound means £1 is worth less compared to other foreign currencies. For example

Jan 2016. £1= $1.50

July 2016 – £1=$1.28

statement 2 is wrong.

Statement 3 not necessary – petrol

Answer – A

Q39. Which one of the following effects of the creation of black money in India has been the main cause of worry to the Government of India?

A. Diversion of resources to the purchase of real estate and investment in luxury housing 

B. Investment in unproductive activities and purchase of precious stones, jewellery, gold, etc. 

C. Large donations to political parties and growth of regionalism

D. Loss of revenue to the State Exchequer due to tax evasion

Tax evasion is the greatest issue.

Answer – D

Q40. Which one of the following is likely to be the most inflationary in its effects?

A. Repayment of public debt

B. Borrowing from the public to finance a budget deficit

C. Borrowing from the banks to finance a budget deficit

D. Creation of new money to finance a budget deficit

If you print more money, the amount of goods doesn’t change. However, if you print money, households will have more cash and more money to spend on goods. If there is more money chasing the same amount of goods, firms will just put up prices.

https://www.economicshelp.org/blog/797/economics/why-printing-money-causes-inflation/

https://www.business-standard.com/article/economy-policy/explained-why-poorer-nations-can-t-just-print-more-money-and-become-rich-118121900185_1.html

Answer – D

Q41. Which one of the following is used in preparing a natural mosquito repellent?

A. Congress grass

B. Elephant grass

C. Lemongrass

D. Nut grass

Answer – C

Lemon grass

Known for its distinct smell, citronella grass (or lemon grass) is the most commonly used natural ingredient in mosquito repellants.

https://www.gardendesign.com/plants/mosquito-repellent.html

https://www.conserve-energy-future.com/is-lemongrass-mosquito-repellent.php

Q42. Consider the following kinds of organisms:

  1. Copepods
  2. Cyanobacteria
  3. Diatoms
  4. Foraminifera

Which of the above are primary producers in the food chains of oceans?

A. 1 and 2

B. 2 and 3

C. 3 and 4

D. 1 and4

Foraminifera are single-celled organisms, members of a phylum or class of amoeboid protists characterized by streaming granular ectoplasm for catching food and other uses; and commonly an external shell of diverse forms and materials. 

Copepods – It is a primary consumer

Diatoms and cyanobacteria are producers

Answer – B

Q43. Consider the following animals:

  1. Hedgehog
  2. Marmot
  3. Pangolin

To reduce the chance of being captured by predators, which of the above organisms roll up and protect its vulnerable parts?

A. 1 and 2

B. 2 only

C. 3 only

D. 1 and 3

Answer – D 

These solitary, primarily nocturnal animals, are easily recognized by their full armor of scales. A startled pangolin will cover its head with its front legs, exposing its scales to any potential predator. If touched or grabbed it will roll up completely into a ball, while the sharp scales on the tail can be used to lash out. 

https://www.worldwildlife.org/species/pangolin

Option 3 – correct

Hedgehogs have a coat of stiff, sharp spines. If attacked they will curl into a prickly and unappetizing ball that deters most predators. 

https://www.nationalgeographic.com/animals/mammals/facts/hedgehog

Option 1 – correct

Q44. With reference to the ‘New York Declaration on Forests’, which of the following statements are correct?

1. It was first endorsed at the United Nations Climate Summit in 2014.

2. It endorses a global timeline to end the loss of forests.

3. It is a legally binding international declaration.

4. It is endorsed by governments, big companies and indigenous communities.

5. India was one of the signatories at its inception.

Select the correct answer using the code given below.

A. 1, 2 and 4

B. 1, 3 and 5

C. 3 and 4

D. 2 and 5

Answer – A

The New York Declaration on Forests is a voluntary and non-legally binding political declaration which grew out of dialogue among governments, companies and civil society, spurred by the United Nations Secretary-General’s Climate Summit held in New York in 2014. The Declaration pledges to halve the rate of deforestation by 2020, to end it by 2030.

Q45. Magnetite particles, suspected to cause neurodegenerative problems, are generated as environmental pollutants from which of the following?

1. Brakes of motor vehicles

2. Engines of motor vehicles

3. Microwave stoves within homes

4. Power plants

5. Telephone lines

Select the correct answer using the code given below:

A. 1, 2, 3 and 5 only

B. 1, 2 and 4 only

C. 3, 4 and 5 only

D. 1, 2, 3, 4 and 5

Answer: D

Magnetic nanoparticles, also called as PM, are a class of nanoparticle that can be manipulated using magnetic fields. Such particles commonly consist of two components, a magnetic material, often iron, nickel and cobalt, and a chemical component that has functionality. They are fine dust emitted by metallurgical industry, vehicle brake systems are the major source of airborne magnetite at the roadside, diesel- and petrol-engine exhaust PM, power plants.

Q46. Which of the following is a filter feeder?

A. Catfish 

B. Octopus

C. Oyster

D. Pelican

Answer: C

Explanation: Filter feeders are a subgroup of suspension feeding animals that feed by straining suspended matter and food particles from water, typically by passing the water over a specialized filtering structure. Some animals that use this method of feeding are clams, krill, sponges, baleen whales, and many fish (including some sharks). Some birds, such as flamingos and certain species of duck, are also filter feeders. 

Although other animals at the reef are said to filter-feed, the method used by sponges is unique to this simple group of animals. In fact, sponges are named because of the many pores covering their bodies. They belong to the Phylum Porifera and may be called poriferans.

https://en.wikipedia.org/wiki/Filter_feeder

https://www.coris.noaa.gov/activities/resourceCD/resources/sponge_lp.pdf

Q47. In case of which one of the following biogeochemical cycles, the weathering of rocks is the main source of release of nutrient to enter the cycle?

A. Carbon cycle

B. Nitrogen cycle

C. Phosphorus cycle

D. Sulphur cycle

Answer: C

Explanation: In nature, phosphorus is found mostly in the form of phosphate ions. Phosphate compounds are found in sedimentary rocks, and as the rocks weather—wear down over long time periods—the phosphorus they contain slowly leaches into surface water and soils. Volcanic ash, aerosols, and mineral dust can also be significant phosphate sources, though phosphorus has no real gas phase, unlike other elements such as carbon, nitrogen, and sulfur.

https://www.britannica.com/science/phosphorus-cycle

Q48. Which of the following are detritivores?

  1. Earthworms
  2. Jellyfish 
  3. Millipedes
  4. Seahorses
  5. Woodlice

Select the correct answer using the codes given below.

A. 1,2 and 4 only

B. 2,3,4 and 5 only

C. 1,3 and 5 only

D. 1,2,3,4 and 5

Answer: C

Explanation: A detritivore is a heterotrophic organism, which obtains its nutrition by feeding on detritus. Detritus is an organic matter made up of dead plant and animal material. Detritivores may also obtain nutrition by coprophagy, which is a feeding strategy involving the consumption of faeces.

Ocean decomposers have a variety of methods for gathering dead material to feed on. Echinoderms like sea urchins, sea stars and sea cucumbers hunt and eat live food, but they also move around and consume decaying organic matter that covers rocks and other surfaces.

Detritivores are often invertebrate insects such as mites, beetles, butterflies and flies; molluscs such as slugs and snails; or soil-dwelling earthworms, millipedes and woodlice. 

Q49. The ‘Common Carbon Metric’ supported by UNEP, has been developed for

A. assessing the carbon footprint of building operations around the world

B. enabling commercial farming entities around the world to enter carbon emission trading

C. enabling governments to assess the overall carbon footprint caused by their countries

D. assessing the overall carbon footprint caused by the use of fossil fuels by the world in a unit time.

Answer: A

Explanation: The Common Carbon Metric is the calculation used to define the measurement, reporting, and verification for GHG emissions associated with the operation of building types of particular climate regions. It does not include value-based interpretation of the measurements such as weightings or benchmarking.

https://c2e2.unepdtu.org/wp-content/uploads/sites/3/2016/09/unep-sbci-common-carbon-metric.pdf

Q50.  Which of the following have species that can establish symbiotic relationship with other organisms?

  1. Cnidarians
  2. Fungi
  3. Protozoa

Select the correct answer using the code given below 

A. 1 and 2 only

B. 2 and 3 only

C. 1 and 3 only

D. 1,2 and 3

Answer: D

Explanation: The symbiotic association between the invertebrate phylum Cnidaria (Coelenterate) and the unicellular dinoflagellate algae, called zooxanthellae, is very common. The most well-known relationship is between zooxanthellae and hermatypic, or reef-forming, corals.

Mycorrhizae are symbiotic relationships that form between fungi and plants. The fungi colonize the root system of a host plant, providing increased water and nutrient absorption capabilities while the plant provides the fungus with carbohydrates formed from photosynthesis.

Mutualism, where two species benefit from their interaction. Some scientists believe that these are the only true examples of symbiosis. For example, termites have a mutualistic relationship with protozoa that live in the insect’s gut. The termite benefits from the ability of bacterial symbionts within the protozoa to digest cellulose. The termite itself cannot do this, and without the protozoa, it would not be able to obtain energy from its food (cellulose from the wood it chews and eats).

Q51. Under the Indian Constitution, concentration of wealth violates

A. The Right to Equality

B. Directive Principles of State Policy

C. the Right to Freedom

D. the Concept of Welfare 

Answer: B

Explanation: Article 39 of the Indian Constitution requires the State to direct its policies towards securing for all its citizens the right to an adequate means of livelihood, availability of material resources, and ensuring prevention of concentration of wealth, etc.

https://www.constitutionofindia.net/constitution_of_india/directive_principles_of_state_policy/articles/Article%2039

Q52. What is the position of Right to Property in India?

A. Legal right available to citizens only

B. Legal right available to any person

C. Fundamental Right available to citizens only

D. Neither Fundamental Right nor Legal Right

Answer: A

Explanation: A citizen’s right to own private property is a human right. The state cannot take possession of it without following due procedure and authority of law, the Supreme Court has held in a judgment.

The state cannot trespass into the private property of a citizen and then claim ownership of the land in the name of ‘adverse possession’, the court said.

https://www.thehindu.com/news/national/private-property-is-a-human-right-says-sc/article30551819.ece

Q53. What was the exact Constitutional status of India on 26th January, 1950?

A. A Democratic Republic

B. A Sovereign Democratic Republic 

C. A Sovereign Secular Democratic Republic

D. A Sovereign Secular Socilaist Democratic Republic

Answer: B

Q54.  Constitutional government means

A. A representative government of a nation with federal structure

B. A government whose Head enjoys nominal powers

C. A government whose Head enjoys real powers

D. A government limited by the terms of the Constitution

Answer: D

Q55. With reference to India, the terms ‘Halbi, Ho and Kui’ pertain to

A. Dance forms of Northwest India

B. Musical instruments

C. Prehistoric cave paintings

D. Tribal languages

Answer: D

Explanation: Can be solved by Tikdam. ‘Ho’ is a famous tribal language. 

Q56. Consider the following statements in respect of Bharat Ratna and Padma Awards:

1. Bharat Ratna and Padma Awards are titles under the Article 18(1) of the Constitution

 of India

2. Padma Awards, which were instituted in 1954, were suspended only once.

3. The number of Bharat Ratna Awards is restricted to a maximum of five in a particular   

year.

Which of the above statements are not correct?

A. 1 and 2 only

B. 2 and 3 only

C. 1 and 3 only

D. 1, 2 and 3

Answer: D

Explanations: The Bharat Ratna is the highest civilian award of the Republic of India. Instituted on 2 January 1954, the award is conferred in recognition of exceptional service/performance of the highest order, without distinction of race, occupation, position, or sex. The award was originally limited to achievements in the arts, literature, science, and public services, but the government expanded the criteria to include “any field of human endeavour” in December 2011. The recommendations for the Bharat Ratna are made by the Prime Minister to the President, with a maximum of three nominees being awarded per year. 

The Bharat Ratna, along with other personal civil honours, was briefly suspended from July 1977 to January 1980, during the change in the national government; and for a second time from August 1992 to December 1995, when several public-interest litigations challenged the constitutional validity of the awards. 

Q57. Consider the following statements:

  1. Statement 1: The United Nations Capital Development Fund (UNCDF) and the Arbor Day Foundation have recently recognized Hyderabad as 2020 Tree City of the World
  2. Statement 2: Hyderabad was selected for the recognition for a year following its commitment to grow and maintain the urban forests.

Which one of the following is correct in respect of the above statements?

A. Both Statement 1 and Statement 2 are correct and Statement 2 is the correct explanation for Statement 1

B. Both Statement 1 and Statement 2 are correct but Statement 2 is not the correct explanation for Statement 1

C. Statement 1 is correct but Statement 2 is not correct

D. Statement 1 is not correct but Statement 2 is correct

Answer: D

Explanation: In a unique distinction, Hyderabad has become the only city in India to be recognised as a ‘Tree City of the World’ by the Arbor Day Foundation and the Food and Agriculture Organization (FAO) of the United Nations. Hyderabad is placed alongside 119 other cities from 63 countries.

The countries have been recognised for their commitment to growing and maintaining urban forests in building healthy, resilient and happy cities.

Q58. Consider the following statements in respect of the Laureus World Sports Award which was instituted in the year 2000:

1. American golfer Tiger Woods was the first winner of this award.

2. The award was received mostly by ‘Formula One’ players so far.

3. Roger Federer received this award maximum number of times compared to others.

Which of the above statements are correct?

A. 1 and 2 only

B. 2 and 3 only

C. 1 and 3 only

D. 1, 2 and 3

Answer: C

Explanation: Swiss tennis player Roger Federer holds the record for the most awards with six, five for Sportsman of the Year and one for Comeback of the Year. 

The inaugural winner of the award was the American golfer Tiger Woods who finished the 1999 season with eight wins, a feat not achieved since 1974, including the PGA Championship.

Formula One has won 4 awards while tennis has a maximum of 11 awards. 

Q59. Consider the following statements in respect of the 32nd Summer Olympics:

1. The official motto for this Olympics is ‘A New World’.

2. Sport Climbing, Surfing, Skateboarding, Karate and Baseball are included in this Olympics.

Which of the above statements is/are correct?

A. 1 only

B. 2 only

C. Both 1 and 2

D. Neither 1 nor 2

Answer: D

Explanation:  The disciplines added by the Japanese Olympic Committee were baseball and softball, karate, sport climbing, surfing and skateboarding, the last four of which made their Olympic debuts, and the last three of which will remain on the Olympic program.

Q60. Consider the following statements in respect of the ICC World Test Championship:

1. The finalists were decided by the number of matches they won.

2. New Zealand was ranked ahead of England because it won more matches than England.

Which of the above statements is/are correct?

A. 1 only

B. 2 only

C. Both 1 and 2

D. Neither 1 nor 2

Answer: D

Q61. Consider the following statements:

1. ‘Right to the City’ is an agreed human right and the UN-Habitat monitors the commitments made by each country in this regard.

2. ‘Right to the City’ gives every occupant of the city the right to reclaim public spaces and public participation in the city.

3. ‘Right to the City’ means that the State cannot deny any public service or facility to the unauthorized colonies in the city.

Which of the statements given above is/are correct?

A. 1 only

B. 3 only

C. 1 and 2

D. 2 and 3

Answer: A

Explanation: 

Right to the city has gained international recognition in the last years could be seen in the United Nations’ Habitat III process, and how the New Urban Agenda (2016) recognized the concept as the vision of “cities for all”

The right to the city is far more than the individual liberty to access urban resources: it is a right to change ourselves by changing the city. It is, moreover, a common rather than an individual right since this transformation inevitably depends upon the exercise of a collective power to reshape the processes of urbanization.

Q62. With reference to India, consider the following statements:

1. Judicial custody means an accused is in the custody of the concerned magistrate and such accused is locked up in a police station, not in jail.

2. During judicial custody, the police officer in charge of the case is not allowed to interrogate the suspect without the approval of the court.

Which of the statements given above is/are correct?

A. 1 only

B. 2 only

C. Both 1 and 2

D. Neither 1 nor 2

Answer: B

Explanation:

Police Custody means that police has the physical custody of the accused while Judicial Custody means an accused is in the custody of the concerned Magistrate. In the former, the accused is lodged in a police station lockup while in the latter, it is the jail.

During Judicial Custody,  the police officer in charge of the case is not allowed to interrogate the suspect. However, the court may allow the interrogations to be conducted if it opines the interrogation being necessary under the facts produced before the court.

Source: https://districts.ecourts.gov.in/sites/default/files/fct.pdf

Q63: With reference to India, consider the following statements:

1. When a prisoner makes out a sufficient case, parole because it becomes a matter of his/her right.

2. State Governments have their own Prisoners Release on Parole Rules.

Which of the statements given above is/are correct?

A. 1 only

B. 2 only

C. Both 1 and 2

D. Neither 1 nor 2

Answer: B

Explanation:

Furlough is seen as a matter of right, to be granted periodically irrespective of any reason and merely to enable the prisoner to retain family and social ties, parole is not a matter of right and may be denied to a prisoner even when he makes out a sufficient case.

Prisons are a State subject: each state has different rules and regulations for it.

Q64. At the national level, which ministry is the nodal agency to ensure effective implementation of the Scheduled Tribes and Other Traditional Forest Dwellers (Recognition of Forest Rights) Act, 2006?

A. Ministry of Environment, Forest and Climate Change

B. Ministry of Panchayat raj

C. Ministry of Rural Development

D. Ministry of Tribal Affairs

Answer – D:

Q65. A legislation which confers on the executive or administrative authority an unguided and uncontrolled discretionary power in the matter of application of law violates which one of the following Articles of the Constitution of India?

A. Article 14

B. Article 28

C. Article 32

D. Article 44

Answer: A

Article 14 of the Constitution of India provides for equality before the law or equal protection of the laws within the territory of India. … “The State shall not deny to any person equality before the law or the equal protection of the laws within the territory of India.” 

Q66: Which one of the following in Indian polity is an essential feature that indicates that it is federal in character?

A. The independence of the judiciary is safeguard.

B. The Union Legislature has elected representatives from constituent units.

C. The Union Cabinet can have elected representatives from regional parties.

D. The Fundamental Rights are enforceable by Courts of Law.

Answer: A

Q67:  Which one of the following best defines the term ‘State’?

A. A community of persons permanently occupying a definite territory independent of external control and possessing an organized government.

B. A politically organized people of a definite territory and possessing an authority to govern them, maintain law and order, protect their natural rights and safeguard their means of sustenance.

C. A number of persons who have been living in a definite territory for a very long time with their own culture, tradition and government.

D. A society permanently living in a definite territory with a central authority, and executive responsible to the central authority and an independent judiciary.

Answer: A

As used in political science, the word state means a community or society politically organized under one independent government within a definite territory and subject to no outside control.

Source: https://www.iilsindia.com/study-material/528569_1600581365.pdf

Q68: With reference to Indian judiciary, consider the following statements:

1. Any retired judge of the Supreme Court of India can be called back to sit and act as a Supreme Court judge by the Chief Justice of India with prior permission of the President of India.

2. A High Court in India has the power to review its own judgement as the Supreme Court does.

Which of the statements given above is/are correct?

A. 1 only

B. 2 only

C. Both 1 and 2

D. Neither 1 nor 2

Answer: C

Explanation: Being a Court of record, the High Court is vested with powers to proceed under Article 226 of the Constitution of India itself and review a judgment,

Q69: With reference to India, consider the following statements:

1. There is only one citizenship and one domicile.

2. A citizen by birth only can become the Head of State.

3. A foreigner once granted citizenship cannot be deprived of it under any circumstances.

Which of the statements given above is/are correct?

A. 1 only

B. 2 only

C. 1 and 3

D. 2 and 3

Answer: A

Explanation: Citizen by Naturalization can also become Head of the state in India but not in USA

Deprivation is a compulsory termination of citizenship of India. A citizen of India by naturalization, registration, domicile and residence, may be deprived of his citizenship by an order of the Central Government if it is satisfied that:

The citizen has obtained citizenship by means of fraud, false representation or concealment of any material fact;

The citizen has shown disloyalty to the Constitution of India;

The citizen has unlawfully traded or communicated with the enemy during a war;

The citizen has, within five years after registration or neutralization, been imprisoned in any country for two years;

The citizen has been ordinarily resident out of India for seven years continuously.

Q70: Which one of the following factors constitutes the best safeguard of liberty in a liberal democracy?

A. A committed judiciary

B. Centralization of powers

C. Elected government

D. Separation of powers

Ans: D

Explanation: Centralization of powers gives rise to monarchy. Therefore decentralization of power is required for the safeguard of liberty. Powers should be divided into central, provincial and local self-governments. This decentralization of powers leads to efficient administration. And people also start cooperating with the functioning of the government.

Q71:  The vegetation of savannah consists of grassland with scattered small trees, but extensive areas have no trees. The forest development in such areas is generally kept in check by one or more or a combination of some conditions.

Which of the following are such conditions?

1. Burrowing animals and termites

2. Fire

3. Grazing herbivores

4. Seasonal rainfall

5. Soil properties

Select the correct answer using the code given below.

A. 1 and 2

B. 4 and 5

C. 2, 3 and 4

D. 1, 3 and 5

Answer: Debatable

Q72: With reference to the water on the planet Earth, consider the following statements:

1. The amount of water in the rivers and lakes is more than the amount of groundwater.

2. The amount of water in polar ice caps and glaciers is more than the amount of groundwater.

Which of the statements given above is/are correct?

A. 1 only

B. 2 only

C. Both 1 and 2

D. Neither 1 nor 2

Answer: B

Explanation:

Distribution of water on Planet Earth:

Streams and Rivers > Atmosphere > Soil Moisture > Lakes > Groundwater > Ice Caps and Glaciers > Oceans

Q73: Consider the following statements:

1. Moringa (drumstick tree) is a leguminous evergreen tree.

2. Tamarind tree is endemic to South Asia.

3. In India, most of the tamarind is collected as minor forest produce.

4. India exports tamarind and seeds of moringa.

5. Seeds of moringa and tamarind can be used in the production of biofuels.

Which of the statements given above are correct?

A. 1, 2, 4 and 5

B. 3, 4 and 5

C. 1, 3 and 4

D. 1, 2, 3 and 5

Ans: B

Statement 1 is incorrect: Moringa (botanical name Moringa oleifera) is a fast-growing, drought-resistant tree of the family Moringaceae, native to the Indian subcontinent. 

Statement 3 is correct: The Forest Rights Act confers ownership rights on minor forest produce like tamarind on tribal populations and residents of the particular forests. Today, about 90 per cent of the tamarind produced in the country comes from the forests.

Statement 4: India is the largest producer of moringa. So we can assume the option is correct.

Statement 5 is correct

Source: https://www.downtoearth.org.in/coverage/agriculture/sweet-n-sour-55668

Q74. The black cotton soil of India has been formed due to the weathering of

A. brown forest soil

B. fissure volcanic rock

C. granite and schist

D. shale and limestone

Answer: B

The Black soil are so called because of their black colouration and derived from the Basalt rock under semi-arid conditions. It is also known as ‘Regur’ (from the Telugu word Reguda) or black cotton soil as it is best suited for cotton cultivation. In India black soil are largely found over Deccan trap region of the states of Maharshtra, Madhya Pradesh, parts of Andhra Pradesh, Northern part of Karnataka, Gujarat, parts of Tamil Nadu and Rajasthan. Several theories have been put forward regarding the origin of this group of soils but most pedologists believe that these soils have been formed due to the solidification of lava spread over large areas during volcanic activity in the Deccan Plateau, in the Triassic Period. 

Q75. With reference to recent developments regarding ‘Recombinant Vector Vaccines’, consider the following statements:

1. Genetic engineering is applied in the development of three vaccines.

2. Bacteria and viruses are used as vectors.

Which of the statements given above is/are correct?

A. 1 only

B. 2 only

C. Both 1 and 2

D. Neither 1 nor 2

Answer: C

The use of recombinant proteins allows the targeting of immune responses focused against few protective antigens. There are a variety of expression systems with different advantages, allowing the production of large quantities of proteins depending on the required characteristics. Live recombinant bacteria or viral vectors effectively stimulate the immune system as in natural infections and have intrinsic adjuvant properties. DNA vaccines, which consist of non-replicating plasmids, can induce strong long-term cellular immune responses.

Q76. In the context of hereditary disease, consider the following statements:

1. Passing on mitochondrial diseases from parent to child can be prevented by mitochondrial replacement therapy either before or after in vitro fertilization of the egg.

2. A child inherits mitochondrial diseases entirely from mother and not from father.

Which of the statements given above is/are correct?

A. 1 only

B. 2 only

C. Both 1 and 2

D. Neither 1 nor 2

Answer: C

Both Statement 1 and 2 are correct.

Mitochondrial replacement therapy(MRT), sometimes called mitochondrial donation, is the replacement of mitochondria in one or more cells to prevent or ameliorate disease. 

MRT originated as a special form of in vitro fertilisation in which some or all of the future baby’s mitochondrial DNA (mtDNA) comes from a third party. This technique is used in cases when mothers carry genes for mitochondrial diseases. 

In Pronuclear transfer technique MRT is done after In vitro fertilisation (IVF).  Hence Statement 1 is true. 

A child inherits mitochondrial diseases only from Mother. Hence,  Statement 2 is true. 

Q77. Bollgard I and Bollgard II technologies are mentioned in the context of

A. Clonal propagation of crop plants

B. Developing genetically modified crop plants

C. Production of plant growth substances

D. Production of biofertilizers

Answer:  B.

Bollgard-1 and 2 are GM crops. 

Bollgard Bt cotton (single-gene technology) is India’s first biotech crop technology approved for commercialization in India in 2002, followed by Bollgard II- a double-gene technology in mid-2006, by the Genetic Engineering Approval Committee (GEAC) in India. 

Q78. In a pressure cooker, the temperature at which the food is cooked depends mainly upon which of the following?

1. Area of the hole in the lid

2. Temperature of the flame

3. Weight of the lid

Select the correct answer using the code given below. 

A. 1 and 2 only

B. 2 and 3 only

C. 1 and 3 only

D. 1,2 and 3 

Answer:  C.

In pressure cooker Technology, the temperature at which food is cooked is determined by the pressure inside it as volume is constant and pressure is regulated by the area of hole in the lid and weight of the lid.  The temperature of the flame doesn’t affect the temperature at which rice will be cooked. Hence, Statements 1 and 3 are true, while statement-2 is not true.  

Q79. Consider the following:

1. Bacteria

2. Fungi

3. Virus

Which of the above can be cultured in an artificial/synthetic medium?

A. 1 and 2 only

B. 2 and 3 only

C. 1 and 3 only

D. 1,2 and 3

Answer: A.   

Viruses replicate only within living cells be it bacteria, fungus or other living organisms. However, bacteria and fungus can be cultured in artificial mediums in laboratory conditions. 

Q80. . Consider the following statements :

1. Adenoviruses have single- stranded DNA genomes whereas retroviruses have double-stranded DNA genomes.

2. Common cold is sometimes caused by an adenovirus whereas AIDS is caused by a retrovirus.

Which of the statements given above is/are correct?

A. 1 only

B. 2 only

C. Both 1 and 2

D. Neither 1 nor 2

Answer:  B

The genome of adenovirus is double-stranded DNA, while the genome of a retrovirus is single-stranded RNA. Both types of viruses have been news over Corona Virus and development of Vaccines.  Hence, Statement-1 is not true. 

Common cold is caused by an adenovirus and other types of viruses, while AIDS is caused by the human immunodeficiency virus (HIV) – a subgroup of retrovirus. Hence, Statement-2 is true. 

Q81.  How is Permaculture farming different from conventional chemical farming?

Permaculture farming discourages monocultural practices but in conventional chemical farming, monoculture practices are predominant. 

Conventional chemical farming can cause increase in soul salinity but the occurrence of such phenomenon is not observed in permaculture farming

Conventional chemical farming is easily possible in semi-arid regions, but permaculture is not so easy in such regions. 

Practice of mulching is very important in permaculture farming, but not necessarily so in Conventional chemical farming.

Select the correct answer using the codes given below

A. 1 and 3 

B. 1,2 and 4

C. 4 only

D. 2 and 3 

Answer:   B

Since the principles of permaculture discourage monoculture, it opens up the opportunity for growing a wide variety of grains, fruits and vegetables, and widens one’s food basket.  It can be possible in all kinds of climate including semi- arid regions.  Hence, Statement-3 is not correct.

Permaculture cultivation discourages monoculture practices, Improves soil fertility and Encourages mulching.  Hence, Statements 1, 2 and 4 are true. 

https://www.downtoearth.org.in/coverage/environment/new-civilisation-primitive-wisdom-59125

Q82.  With reference to ‘palm oil, consider the following statements:

1. The palm oil tree is native to SouthEast Asia.

2. Palm oil is a raw material for some industries producing lipstick and perfumes.

3. The palm oil can be used to produce biodiesel 

Which of the statements given above are correct?

A. 1 and 2 only

B. 2 and 3 only

C. 1 and 3 only

D. 1, 2 and 3 only

Answer:   B

Palm oil is native to Africa and brought to southeast Asia during colonial times, palm oil plantations now cover huge areas of Malaysia and Indonesia.  

Palm oil is used in lipstick as it holds color well, doesn’t melt at high temperatures, and has a smooth application and virtually no taste.

Palm can be used for making biodiesel also. 

Hence, Statement-1 is not true while Statement-2 and 3 are true.

Q83. With reference to the Indus River system, of the following four rivers, three of them pour into one of them which joins the Indus directly. Among the following, Which one is such a river that joins the Indus directly?

A. Chenab

B. Jhelum

C. Ravi

D. Sutlej  

Answer:  D

About 17 kilometres north of Uch Sharif in Pakistan, the Sutlej unites with the Chenab river which already united with Jhelum and Ravi river, forming the Panjnad River. It finally flows into the Indus river about 100 kilometres west of the city of Bahawalpur in Pakistan. 

Q84.  With reference to India, Didwana, Kuchaman, Sargol and Khatu are the names of

A. glaciers

B. mangrove areas

C. Ramsar sites

D. saline lakes

Answer: D

All of them are saline lakes. 

Q85.  Consider the following rivers: 

  1. Brahmani
  2. Nagavali 
  3. Subarnarekha 
  4. Vamshandhra

Which of the above rise from Eastern Ghats? 

A. 1 and 2 

B. 2 and 4

C. 3 and 4 

D. 1 and 3

Answer:   B

Statement-1 and 3 are incorrect, while 2 and 4 are correct. 

Brahmani and Subarnarekha originate from central Indian Plateau regions. Vamsadhara River is an east-flowing river which originates in Kalahandi district of Odisha ie. Eastern Ghats.  The River Nagavali also known as Langulya is one of the main rivers of Southern Odisha and Northern Andhra Pradesh States in India, between Rushikulya and Godavari basins. 

Q86.  Consider the following statements: 

  1. The Global Ocean Commission grants licences for seabed exploration and mining in international waters. 
  2. India has received licences for seabed mineral exploration in international waters.
  3. ‘Rare earth minerals’ are present on the seafloor in international waters. 

Which of the statements given above are correct?

A. 1 and 2 only

B. 2 and 3 only

C. 1 and 3 only

D. 1, 2 and 3 ony

Answer: B

International Seabed Authority (ISA) set up under UNCLOS focuses mainly on organising, regulating and controlling all the mineral-related activities in the international seabed area that are beyond the limits of national jurisdiction. Hence, Statement-1 is not correct. 

India was the first country to receive the status of a ‘Pioneer Investor ‘ in 1987 and was given an area of about 1.5 lakh sq km in the Central Indian Ocean Basin (CIOB) for nodule exploration. In 2002, India signed a contract with the ISA and after complete resource analysis of the seabed 50% was surrendered and the country retained an area of 75,000 sq km. Ocean floors are rich sources of Rare Earth Minerals along with heavy metals.  

Hence, Statement-2 and 3 are correct. 

Q87. Among the following, which one of the least water- efficient crops? 

A. Sugarcane

B. Sunflower

C. Pearl millet

D. Red gram  

Answer: A .

Sugarcane is the least water efficient crop as evident from Droughts in Marathwada a few years ago. 

Q88. Consider the following Statements: 

  1. In the tropical zone, the western sections of the oceans are warmer than the eastern sections owing to the influence of the trade winds. 
  2. In the temperate zone, westerlies make the eastern sections of oceans warmer than the western sections. 

Which of the statements given above is/are correct?

A. 1 only

B. 2 only

C. Both 1 and 2 only

D. Neither 1 nor 2

Answer:  C. 

Normally, sea surface temperature is Western parts of the ocean rather than the eastern part. This is due to the trade winds blowing from east to west  carry warm surface water from east to west and allow upwelling of cold water in eastern coastal areas. Hence, Statement-1 is true. 

In the temperate zone,  Westerlies flowing from west to east bring warm water in eastern temperate regions; hence, temperature is slightly higher than eastern part. Hence, Statement-2 is true. 

Q89.  In the context of India’s preparation for Climate- Smart Agriculture, Consider the following statements: 

1.  The ‘ Climate- Smart Village’ approach in India is a part of a project led by the Climate Change, Agriculture and Food Security (CCAFS), an international research programme. 

2. The project CCAFS is carried out under Consultative Group on International Agricultural Research(CGIAR) headquartered in France. 

3. The international Crops Research Institute for the Semi- Arid Tropics (ICRISAT) in India is one of the CGIAR’s research centres. 

Which of the statements given above are correct?

A. 1 and 2 only

B. 2 and 3 only

C. 1 and 3 only

D. 1,2 and 3 

Answer: D

CCAFS project’s major activities in India include test, evaluate and develop portfolios of climate-smart interventions for different agro-ecological zones and farm types; promotion of Climate Smart Agriculture (CSAT) through the Climate-Smart Village (CSV) approach; weather-based insurance; use of ICT for dissemination of climate information based agro-advisories. 

CCAFS is carried out under CGIAR headquartered In France and ICRISAT located in Patancheru, Telangana is one of the CGIAR’s research centres. Hence, All the statements are true. 

Q90.  “ Leaf litter decomposes faster than in any other biome and as a result the soil surface is often almost bare. Apart from trees, the vegetation is largely composed of plant forms that reach up into the canopy vicariously, by climbing the trees or growing as epiphytes, rooted on the upper branches of trees”. This is the most likely description of 

A. Coniferous forest

B. Dry deciduous forests

C. Mangore forest

D. Tropical  rainforest

Answer:  D

All the characteristics mentioned in the question are of tropical rainforest. 

The hot and humid conditions lead to faster decay and absorption of nutrients.  Trees are only 70% of plants, the rest are creepers and climbers that reach up into the canopy and compete for water and sunlight. 

Q91. Water can dissolve more substances than any other liquid because:

A. It is Polar in nature

B. It is a good conductor of heat

C. It has a high value of specific heat

D. It is an oxide of hydrogen

Answer – A (polarity of water)

Source:- https://www.usgs.gov/special-topic/water-science-school/science/water-qa-why-water-universal-solvent?qt-science_center_objects=0#qt-science_center_objects

Q92. With Reference to street lighting, how are sodium lamps different from LED Lamps?

  1. Sodium lamp produces light in 360 degrees but it is not so in the case of LED lamps
  2. As Street lights, sodium lamps have a longer lifespan than street lamps.
  3. The spectrum of visible light from sodium lamps is almost monochromatic while LED lamps offer significant colour advantages in street lightning

Select the correct answer from the code given below

A. 3 only

B. 2 only

C. 1 and 3 only

D. 1, 2 and 3 only

Answer:  C

What’s the Major Upside to LED Lights?

There are four major advantages to LED lighting:

LEDs have an extremely long lifespan relative to every other lighting technology. 

New LEDs can last 50,000 to 100,000 hours or more. The typical lifespan for a fluorescent bulb, by comparison, is 10-25% as long at best (roughly 10,000 hours).

LEDs are extremely energy efficient relative to every other commercially available lighting technology. There are several reasons for this to include the fact they waste very little energy in the form of infrared radiation (much different than most conventional lights to include fluorescent lights), and they emit light directionally (over 180 degrees versus 360 degrees which means there are far fewer losses from the need to redirect or reflect light).

Very high light quality.

Very low maintenance costs and hassle

Sodium vapour lights have the worst colour rendering on the market. LPS lamps in particular are monochromatic which means that the objects illuminated by them appear shadowy black rather than the colour you would see during daylight. HPS lamps are better but are still surpassed by virtually every other light on the market.  

https://www.stouchlighting.com/blog/led-vs-hps-lps-high-and-low-pressure-sodium

https://www.energy.gov/energysaver/led-lighting

Q93. The term ACE-2 is talked about in the context of?

A. Genes introduced in the genetically modified crops

B. Development of India’s own navigation satellite system

C. Radio collar for wildlife trafficking

D. Spread of viral diseases

Answer – D

Solution:- 

The SARS-CoV-2 virus enters a human cell through ‘ACE2 receptors’, which are enzymes that generate small proteins that are useful for the cell. The coronaviruses have horn-like projections, called spike proteins. The spike proteins bind to ACE2, like a key inserted into a lock, and the virus gains entry into the cell and infects it. 

Source The Hindu https://www.google.com/amp/s/www.thehindubusinessline.com/business-tech/ace2-receptors-how-the-sars-cov-2-virus-transmits-so-fast/article33956645.ece/amp/

Q94. Bisphenol A(BPA), a cause of concern is a structural/key component in the manufacture of which of the following kinds of plastic?

A. Low-density Polyethylene

B. Polycarbonate

C. Polyethylene terephthalate

D. polyvinyl chloride

Answer- B

Solution:-  In the news because BPA mimics estrogen and was found in feeding bottles of infants. It is considered carcinogenic and linked with lower IQ in newborns

Source:- although it is an old news, yet every time you look at any water bottle it says BPA free.

https://www.google.com/amp/s/indianexpress.com/article/india/study-shows-use-of-banned-synthetic-in-feeding-bottles-5741631/lite/

Q95. Triclosan is considered harmful when exposed to. high levels for a very long time, is most likely present in which of the following?

A. food preservatives

B. Food ripening substance

C. Reused plastic containers 

D. Toiletries

Solution:- D 

In news due to Covid hand washing protocols. A study by IIT HYDERABAD found its harmful uses. Common sense discussion – the killing of good bacteria due to frequent hand washing

Source:- 

https://m.timesofindia.com/city/hyderabad/triclosan-in-soap-toothpaste-harmful-to-health-iit-h/amp_articleshow/79747418.cms

Q96. Which one of the following is the reason why astronomical distances are measured in light years?

A. Distance among stellar bodies do not change

B. Gravity of stellar bodies do not change

C. Light always travels in a straight line

D. Speed of light is always the same

Answer: D

Solution:- The only thing absolute in this universe according to Einstein is the speed of light, rest everything is relative. So to get an idea of interstellar distance, Light Year is the best method, as it is really large and appropriate for such lengths and is even concise, as no one would remember all the digits or exponents when the distance is expressed in Kilometres.

Q97. We adopted our parliamentary democracy based on the British model,  but how does our model differ from the British model?

  1. As Regards legislation the British parliament is Supreme or sovereign, but in India the power of the parliament to legislate is limited.
  2. In India matters related to the constitutionality of an amendment of an act of the parliament are referred to the constitutional bench by the Supreme Court

Select the correct answer using the code given below:

A. 1 only

B. 2 only 

C. Both 1 and 2

D. Neither 1 nor 2

Answer: C

Solution :- Laxmikant chapter Parliamentary System last page – Distinction between indian and British Models

Q98. With reference to the union government consider the following statement

  1. N Gopalaswamy Ayengar Committee suggested that a minister and a Secretary be designated solely for pursuing the subject of administrative reform and promoting it.
  2. In 1970 the Department of Personnel was constituted on the recommendation of the Administrative Reforms Commission 1966 and this was placed under the Prime Minister’s charge.

Which of the following statements given above is/are correct?

A. 1 only

B. 2 only

C. Both 1 and 2

D. Neither 1 nor 2

Answer: B

Source :- https://darpg.gov.in/about-department-0

Q99. Right to Privacy is protected under which article of the constitution of India?

A. Art 15

B. Art 19

C. Art 21

D. Art 29

Answer:- C

Direct and repeated question. Justice K S Puttaswamy judgement of supreme court asked before in main exam as well

Source:- https://www.civilsdaily.com/news/surveillance-and-human-right/

Q100. Consider the following statement

  1. In India there is no law restricting the candidate from contesting in one Lok Sabha election from 3 constituencies
  2. In 1991 lok sabha  elections Sri Devi Lal contested from 3 lok sabha constituencies
  3. As per the existing rules if a  candidate contests  in one Lok Sabha elections from many constituencies, his/her party  should bear the cost of bye elections vacated by him or her in the event of him/her winning in all the constituencies

Which of the statements given above is/ are correct? 

A. 1 only

B. 2 only

C. 1 and 3 only

D. 2 and 3 only

Answer- A 

Solution :- Laxmikant Electoral Reform page 71.3 contestants restricted to two constituencies

Subscribe
Notify of
0 Comments
Inline Feedbacks
View all comments

JOIN THE COMMUNITY

Join us across Social Media platforms.

💥Mentorship New Batch Launch
💥Mentorship New Batch Launch